Diktat Pelatihan Olimp Matematika.pdf

  • Uploaded by: Sukrya Rya
  • 0
  • 0
  • October 2019
  • PDF

This document was uploaded by user and they confirmed that they have the permission to share it. If you are author or own the copyright of this book, please report to us by using this DMCA report form. Report DMCA


Overview

Download & View Diktat Pelatihan Olimp Matematika.pdf as PDF for free.

More details

  • Words: 21,995
  • Pages: 71
(a) a + b = b + a, (b) ab = ba. 3. Eksistensi unsur identitas, yaitu (a) Terdapat 0 di R yang memenuhi a+0=a untuk semua a di R. (b) Terdapat 1 di R dan 1 = 0 yang memenuhi a1 = a untuk semua a di R. 4. Eksistensi unsur invers, yaitu (a) Untuk masing-masing a di R terdapat −a di R yang memenuhi a + (−a) = 0. (b) Untuk masing-masing a di R yang tidak nol terdapat a−1 di R yang memenuhi aa−1 = 1. 5. Sifat distributif, yaitu untuk setiap a, b, c di R berlaku a(b + c) = ab + ac. Sebagai kosekuensi dari sifat-sifat pada aksioma lapangan di atas, diperoleh sifat-sifat berikut (yang dapat dianggap sebagai teorema). 1. −a dan a−1 yang memenuhi sifat 4 di atas adalah tunggal. 2. 0a = 0, untuk setiap a di R. 3. (−1)a = −a, untuk setiap a di R. 4. −(−a) = a, untuk setiap a di R. 5. (−a)(−b) = ab, untuk setiap a di R. −1  = a, untuk setiap a di R yang tidak nol. 6. a−1 Sebagai suatu teorema, sifat-sifat di atas harus dibuktikan. Berikut diberikan beberapa contoh pembuktian.

2

Bukti untuk 2. Ambil a ∈ R sebarang. Dengan menggunakan sifat identitas, invers, asosiatif, dan distributif diperoleh 0a = 0 + 0a = (−0a + 0a) + 0a = −0a + (0a + 0a) = −0a + (0 + 0)a = −0a + 0a = 0. Karena a ∈ R sebarang, maka 0a = 0, untuk setiap a di R. Bukti untuk 3. Ambil a ∈ R sebarang. Dengan menggunakan sifat identitas, invers, asosiatif, distributif, dan sifat 2 di atas diperoleh (−1)a = 0 + (−1)a = (−a + a) + (−1)a = −a + (a + (−1)a) = −a + (1a + (−1)a) = −a + (1 + (−1))a = −a + 0a = −a + 0 = −a. Karena a ∈ R sebarang, maka (−1)a = −a, untuk setiap a di R. Bukti keempat sifat yang lain digunakan sebagai latihan.

1.2

Aksioma Urutan

Terdapat himpunan P yang merupakan himpunan bagian dari himpunan bilangan real yang memenuhi tiga sifat berikut. (1) Untuk bilangan real x sebarang, berlaku salah satu dari: (i) x = 0, atau (ii) x ∈ P , atau (iii) −x ∈ P . (2) Jika x, y ∈ P , maka x + y ∈ P . (3) Jika x, y ∈ P , maka xy ∈ P . Sifat (1), (2), dan (3) di atas berturut-turut disebut sifat trikotomi, sifat ketertutupan operasi tambah, dan sifat ketertutupan operasi kali. Himpunan P di atas disebut himpunan bilangan real positif. Berikutnya ada kesepakatan bahwa notasi x > 0 digunakan jika x ∈ P . Dengan demikian ketiga sifat di atas dapat ditulis ulang sebagai berikut. (1’) Untuk bilangan real x sebarang, berlaku salah satu dari: (i) x = 0, atau (ii) x > 0, atau (iii) −x > 0. 3

(2’) Jika x > 0 dan y > 0, maka x + y > 0. (3’) Jika x > 0 dan y > 0, maka xy > 0. Berikut didefinisikan relasi ”lebih besar dari” dan relasi ”lebih kecil dari” untuk dua bilangan real. x dikatakan lebih besar dari y, dinotasikan x > y, jika x − y > 0. x dikatakan lebih kecil dari y, dinotasikan x < y, jika y − x > 0. Dapat ditunjukkan bahwa x > y ekivalen dengan y < x. Notasi x ≥ y, dibaca x lebih besar dari atau sama dengan y, digunakan jika x > y atau x = y. Konsekuensi dari sifat-sifat di atas, diperoleh sifat-sifat berikut (yang dapat dianggap sebagai teorema). 1. Untuk setiap pasang bilangan real a dan b pasti berlaku salah satu dari a < b, atau a = b, atau a > b. 2. Jika a < b dan b < c, maka a < c. 3. Jika a < b, maka a + c < b + c. 4. Jika a < b dan c > 0, maka ac < bc. 5. Jika a > 0 dan b > 0, maka ab > 0. 6. Jika a < b dan c < 0, maka ac > bc. 7. Untuk setiap a di R berlaku a2 ≥ 0. Selanjutnya, a2 = 0 jika dan hanya jika a = 0. 8. Jika a > b > 0, maka

1 a

< 1b .

9. Jika a > b > 0 dan c > d > 0, maka ac > bd dan

a d

> cb .

10. Jika a > b, maka an > bn untuk n bilangan asli ganjil. 11. Jika a > b > 0, maka an > bn untuk n bilangan asli. Bukti sifat-sifat di atas digunakan sebagai latihan.

2

Ketaksamaan

Sifat-sifat yang berkaitan dengan aksioma urutan banyak digunakan pada masalah-masalah yang berkaitan dengan ketaksamaan. Pembahasan ketaksamaan berikut menggunakan pendekatan pemecahan masalah dengan beberapa contoh. Contoh 1 Tunjukkan bahwa a + a1 ≥ 2 untuk setiap bilangan real a > 0, dan akan merupakan kesamaan jika dan hanya jika a = 1. Penyelesaian: Untuk setiap bilangan real a berlaku a2 − 2a + 1 = (a − 1)2 ≥ 0.

4

Sehingga a2 + 1 ≥ 2a. Karena a > 0 maka a+

1 ≥ 2. a

Selanjutnya a+

1 = 2 ⇔ a2 + 1 = 2a ⇔ a2 − 2a + 1 = 0 ⇔ (a − 1)2 = 0 ⇔ a = 1. a

Contoh 2 Misalkan a, b, dan c adalah bilangan real positif dan a + b + c = 1. Tunjukkan bahwa 1 1 1 + + ≥ 9. a b c Penyelesaian: Dari a + b + c = 1 diperoleh 1 a 1 b 1 c

b c + a a c +1+ b b + +1 c

= 1+ a b a c

= =

Sehingga 1 1 1 + + a b c

Menurut Contoh 1,

a b

+

b a

≥ 2,

    a b c c a b + = 1+ + +1+ + + +1 a a b b c c      a c c b a b + + + + + +3 = b a c a b c 

a c

+

c a

≥ 2,

c b

+

b c

≥ 2. Oleh karena itu

1 1 1 + + ≥ 2 + 2 + 2 + 3 = 9. a b c

Contoh 3 Buktikan bahwa jika a dan b bilangan real positif maka  2 a2 + b2 a+b √ ≥ ≥ ab ≥ 1 1 . 2 2 a + b Selanjutnya, ketaksamaan ini akan berlaku sebagai kesamaan jika dan hanya jika a = b. Bukti: Perhatikan bahwa Hal ini ekivalen dengan

a2 − 2ab + b2 = (a − b)2 ≥ 0.

(1)

a2 + b2 ≥ 2ab.

(2)

Ditambah dengan a2 + b2 untuk kedua ruas, Ketaksamaan (2) ekivalen dengan   2 a2 + b2 ≥ a2 + 2ab + b2 = (a + b)2 , 5

yang ekivalen dengan



a+b a2 + b2 ≥ . (3) 2 2 Perhatikan bahwa Ketaksamaan (3) merupakan kesamaan jika dan hanya jika Ketaksamaan (1) merupakan kesamaan. Hal ini terjadi jika dan hanya jika a = b. √ √ Dengan mengganti a dan b pada Ketaksamaan (2) berturut-turut dengan a dan b diperoleh √ a + b ≥ 2 ab, yang ekivalen dengan

a+b √ ≥ ab. 2

(4)

Dengan demikian, Ketaksamaan (4) merupakan kesamaan jika dan hanya jika Ketaksamaan (4) ekivalen dengan 2 1 ≤√ , a+b ab yang ekivalen dengan √ 2 2 1 √ ab = = ab. ab ≤ 1 1 a+b ab a + b



a=



b, atau a = b.

(5)

Selanjutnya ketaksamaan (5) merupakan kesamaan jika dan hanya jika a = b. Dari Ketaksamaan (3), (4), dan (5) diperoleh  a2 + b2 2 a+b √ ≥ ≥ ab ≥ 1 1 . 2 2 a + b Selanjutnya, ketaksamaan di atas akan merupakan kesamaan jika dan hanya jika a = b.  √ 2 2 a+b 2 Untuk bilangan real positif a dan b, a +b ab, dan 1 + 1 pada contoh di atas berturut2 , 2 , a

b

turut disebut rataan kuadrat (QM), rataan aritmatika (AM), rataan geometri (GM), dan rataan harmonik (HM) dari a dan b. Dengan demikian, untuk bilangan real positif a dan b kita mempunyai QM ≥ AM ≥ GM ≥ HM, selanjutnya QM = AM = GM = HM jika dan hanya jika a = b. Rataan kuadrat, rataan aritmatika, rataan geometri, dan rataan harmonik dari n bilangan real positif a1 , a2 , . . . , an berturut-turut adalah  a21 + a22 + . . . + a2n QM = n a1 + a2 + . . . + an AM = , n √ GM = n a1 a2 . . . an , dan n HM = 1 1 1 . a1 + a2 + . . . + an Teorema berikut merupakan perumuman dari Contoh 3. Teorema 1 Jika QM, AM, GM, dan HM berturut-turut menyatakan rataan kuarat, rataan aritmatika, rataan geoetri, dan rataan harmonik dari bilangan real positif a1 , a2 , . . . , an , maka QM ≥ AM ≥ GM ≥ HM . Selanjutnya, ketaksamaan ini akan berlaku sebagai kesamaan jika dan hanya jika a1 = a2 = . . . = an . 6

Contoh 4 Contoh 2 dikerjakan dengan menggunakan ketaksamaan AM-HM. Menurut ketaksamaan AM-HM, kita mempunyai 3

Karena

1 a

+

1 b

+

1 c

> 0 maka

1 a

+

1 b

1 a

+

+

1 c

1 b

+



1 c

a+b+c 1 = . 3 3

≥ 9.

Contoh 5 Misalkan a, b, dan c adalah bilangan real positif yang memenuhi (1+a)(1+b)(1+c) = 8. Buktikan bahwa abc ≤ 1. Selanjutnya tentukan kapan kesamaan terjadi. Penyelesaian: Dari yang diketahui diperoleh 8 = (1 + a)(1 + b)(1 + c) = 1 + (a + b + c) + (ab + bc + ca) + abc.

(6)

Menurut ketaksamaan AM-GM, 1

a + b + c ≥ 3(abc) 3

2

dan ab + bc + ca ≥ 3(abc) 3 ,

(7)

masing-masing ketaksamaan di atas merupakan kesamaan jika dan hanya jika a = b = c. Dari (6) dan (7) diperoleh   1 2 1 3 8 ≥ 1 + 3(abc) 3 + 3(abc) 3 + abc = 1 + (abc) 3 , yang ekivalen dengan

1

1 + (abc) 3 ≤ atau

√ 3

8 = 2,

1

(abc) 3 ≤ 2 − 1 = 1, yaitu abc ≤ 1, dan kesamaan terjadi jika dan hanya jika a = b = c = 1.

3

Sukubanyak (Polinom)

Misalkan F menyatakan sistem bilangan real atau sistem bilangan rasional dan n adalah bilangan bulat tidak negatif. Bentuk f (x) = a0 + a1 x + a2 x2 + · · · + an xn dengan a0 , a1 , a2 , · · · , an di F dan an = 0, disebut sukubanyak atas F berderajad n. Himpunan semua sukubanyak atas F ditandai dengan F [x]. Berikut diberikan beberapa sifat sukubanyak yang sering digunakan. Teorema 2 (Algoritma Pembagian) Misalkan f (x) dan g(x) di F [x] dan g(x) bukan sukubanyak nol. Maka terdapat sukubanyak q(x) dan r(x) di F [x] yang tunggal dan memenuhi f (x) = q(x)g(x) + r(x) dengan r(x) merupakan sukubanyak nol atau r(x) bukan sukubanyak nol yang berderajad kurang dari derajad g(x). 7

Dalam teorema di atas, q(x) disebut hasilbagi dan r(x) disebut sisa pembagian. Selanjutnya jika r(x) merupakan sukubanyak nol maka dikatakan f (x) habis dibagi oleh g(x). Teorema 3 (Teorema Sisa) Jika sukubanyak f (x) dibagi oleh (x − a) maka sisanya adalah f (a). Bilangan a di F disebut akar dari sukubanyak f (x) jika f (a) = 0. Sebagai akibat dari teorema di atas diperoleh teorema berikut. Teorema 4 (Teorema Faktor) Sukubanyak f (x) habis dibagi oleh (x − a) jika dan hanya jika a merupakan akar dari f (x). Contoh 6 Sukubanyak f (x) = 2x3 + Ax2 + x − B habis dibagi oleh (x − 2) dan bersisa -9 jika dibagi (x + 1). Tentukan nilai A dan B. Penyelesaian: Karena f (x) habis dibagi oleh (x − 2) maka 0 = f (2) = 2(2)3 + A(2)2 + 2 − B = 4A − B + 18.

(8)

Karena f (x) bersisa -9 jika dibagi (x + 1) maka −9 = f (−1) = 2(−1)3 + A(−1)2 + (−1) − B = A − B − 3.

(9)

Dengan menyelesaikan sistem persamaan di atas (mengeliminasi (8) dan (9) serta mensubstitusikan ke salah satu darinya) diperoleh A = −4 dan B = 2. Contoh 7 Tentukan semua bilangan asli n sehingga sukubanyak x2 + x + 1 membagi sukubanyak x2n + xn + 1. Penyelesaian: Perhatikan bahwa x3 − 1 = (x − 1)(x2 + x + 1) dan x3 − 1 membagi x3m − 1.

(i) Untuk n = 3k, x2n + xn + 1 = x6k + x3k + 1 = (x6k − 1) + (x3k − 1) + 3 = (x2 + x + 1)Q(x) + 3. (ii) Untuk n = 3k + 1, x2n + xn + 1 = x6k+2 + x3k+1 + 1 = x2 (x6k − 1) + x(x3k − 1) + x2 + x + 1 = (x2 + x + 1)R(x). (iii) Untuk n = 3k + 2, x2n + xn + 1 = = = =

x6k+4 + x3k+2 + 1 x4 (x6k − 1) + x2 (x3k − 1) + x4 + x2 + 1 x4 (x6k − 1) + x2 (x3k − 1) + x(x3 − 1) + x2 + x + 1 (x2 + x + 1)S(x).

8

Jadi x2 + x + 1 membagi x2n + xn + 1 jika dan hanya jika n bukan kelipatan 3. Sifat yang lain dari sukubanyak yang sering digunakan adalah sifat simetri akar, yang lebih dikenal dengan nama Teorema Vieta, yaitu hasil tambah dan hasil tambah dari hasil kali akar-akar suatu sukubanyak. (a) Jika sukubanyak ax2 + bx + c mempunyai akar-akar x1 dan x2 maka ax2 + bx + c = a(x − x1 )(x − x2 ) = ax2 − a(x1 + x2 )x + ax1 x2 . Sehingga x1 + x2 = −

b a

dan x1 x2 =

c . a

(b) Misalkan x1 , x2 , dan x3 akar-akar sukubanyak ax3 + bx2 + cx + d. Dengan ekspansi a(x − x1 )(x − x2 )(x − x3 ) = ax3 − a(x1 + x2 + x3 )x2 + a(x1 x2 + x2 x3 + x3 x1 )x − ax1 x2 x3 dan komparasi koefisien diperoleh b c d x1 + x2 + x3 = − , x1 x2 + x2 x3 + x3 x1 = , dan x1 x2 x3 = − . a a a Contoh 8 Misalkan x1 , x2 , dan x3 akar-akar dari x3 + 3x2 − 7x + 1. Tentukan x21 + x22 + x23 . Penyelesaian: Menurut Teorema Vieta, x1 + x2 + x3 = −3 dan x1 x2 + x2 x3 + x3 x1 = −7. Sehingga 9 = (x1 + x2 + x3 )2 = x21 + x22 + x23 + 2 (x1 x2 + x2 x3 + x3 x1 ) = x21 + x22 + x23 + 2(−7).

Oleh karena itu x21 + x22 + x23 = 23.

4

Sistem Persamaan

Bentuk yang melibatkan variabel, yaitu f (x1, x2 , . . . , xn ) = c disebut persamaan dengan n buah variabel. Sistem persamaan adalah suatu sistem yang terdiri dari dua atau lebih persamaan, yaitu f1 (x1, x2 , . . . , xn ) = c1 f2 (x1, x2 , . . . , xn ) = c2 .. . fm (x1, x2 , . . . , xn ) = cm 9

Sistem persamaan di atas disebut sistem persamaan dengan n buah variabel dan m persamaan. Solusi dari suatu sistem persamaan adalah solusi secara simultan dari semua persamaan di dalam sistem itu. Cara baku untuk mencari solusi suatu sistem persamaan dengan cara eliminasi dan atau substitusi. Berikut akan diberikan beberapa contoh soal yang tidak regular. Contoh 9 Cari semua solusi real dari sistem persamaan 2 x 2 y+ y 2 z+ z

x+

= 2y = 2z = 2x.

Penyelesaian: Misalkan (x, y, z) solusi sistem persamaan di atas. Diantara x, y, dan z tidak mungkin ada yang nol. Perhatikan bahwa jika salah satu positif maka dua yang lain juga positif. Selanjutnya, dengan mengalikan dengan -1 akan diperoleh solusi yang lain. Asumsikan x, y, z > 0. Dengan menggunakan ketaksamaan AM-GM untuk masing-masing persamaan diperoleh

  √ √ 2 2 = 2 2 ⇐⇒ y ≥ 2, 2y = x + ≥ 2 x x x

  √ √ 2 2 = 2 2 ⇐⇒ z ≥ 2, 2z = y + ≥ 2 y y y

  √ √ 2 2 2x = z + ≥ 2 z = 2 2 ⇐⇒ x ≥ 2. z z Dengan menambahkan semua persamaan dari sistem persamaan semula dan hasil di atas, diperoleh   √ √ 2 2 2 ≤ 3 2. 3 2≤x+y+z =2 + + x y z √ √ √ √  Dengan demikan haruslah x = y = z = 2. Selanjutnya dapat ditunjukkan bahwa 2, 2, 2 √ √   √ dan − 2, − 2, − 2 adalah solusi yang dimaksud. Contoh 10 Cari semua solusi real dari sistem persamaan 4x2 4x2 + 1 4y 2 4y 2 + 1 4z 2 4z 2 + 1

= y = z = x.

Penyelesaian: Perhatikan fungsi f : [0, ∞) −→ [0, ∞), f (t) =

4t2 , 4t2 + 1

merupakan fungsi monoton naik murni.Oleh karena itu jika x < y maka y = f (x) < f (y) = z. Akibatnya z = f (y) < f (z) = x. Sehingga x < y < z < x, suatu yang tidak mungkin. Dengan cara 10

yang sama jika x > y maka akan diperoleh suatu kontradiksi. Jadi x = y. Dengan menggunakan argumen yang sama diperoleh y = z. Jadi x = y = z. Dengan menyelesaikan persamaan 4t2 =t 4t2 + 1 t = 0 atau t = 12 . Jadi solusi dari sistem persamaan di atas hanyalah tripel (0, 0, 0) dan diperoleh  1 1 1 2, 2, 2 . Contoh 11 Tentukan semua solusi real dari sistem persamaan x2 + y 2 + z 2 = 1 2 2 x y + y 2 z 2 + z 2 x2 = 9x2 y 2 z 2 Penyelesaian: Dari persamaan pertama, tidak mungkin x = y = z = 0. Dari persamaan kedua tidak mungkin jika satu variabel nol dan dua variabel tidak nol. Kasus I: Jika dua variabel nol dan satu variabel tidak nol. Misalkan x = y = 0, z = 0. Diperoleh z = ±1. Dengan demikian (0, 0, ±1) merupakan solusi. Dengan cara yang sama diperoleh (0, ±1, 0) dan (±1, 0, 0) juga merupakan solusi. Kasus II: Jika ketiga variabel tidak nol. Persamaan kedua ekivalen dengan 1 1 1 + 2 + 2 = 9. 2 x y z Digunakan AM-HM, persamaan pertama, dan persamaan di atas diperoleh 1 x2 + y 2 + z 2 = ≥ 3 3 Jadi

x2 +y 2 +z 2 3

=

3

1 + 12 + 12 x2 y z

3 1 x2

+

1 y2

+

1 z2

=

3 1 = . 9 3

√ √ √ . Oleh karena itu x2 = y 2 = z 2 . Diperoleh (± 13 3, ± 13 3, ± 13 3) solusi

sistem persamaan di atas.

Soal-soal Latihan 1. Diketahui a + b = 1 dan a2 + b2 = 2. Tentukan a4 + b4 . √ √ 2. Sederhanakan 9 + 80 − 9 − 80 (tanpa melibatkan tanda akar). 3. Buktikan bahwa jika a, b, c ∈ R, dan a2 + b2 + c2 = 1 maka − 12 ≤ ab + bc + ca ≤ 1. 4. Tentukan bilangan real a agar hasil tambah kuadrat akar-akar x2 + (a − 2)x − a − 3 minimum.

5. Hitung nk=1 k!(k 2 + k + 1). 6. Tentukan jumlah dari 

1 1 1+ 2 + 2 + 1 2



1 1 1 + 2 + 2 + ··· + 2 3

 1+

1 1 + 20012 20022

7. Diketahui f (x) = 2x + 1 dan g(f (x)) = x2 + 3x + 1. Tentukan g(3). 11

8. Misalkan x dan y bilangan real dan x2 + 3xy + y 2 = 60. Tentukan nilai maksimum yang mungkin untuk xy. 9. Misalkan semua akar dari x6 − 6x5 + ax4 + bx3 + cx2 + dx + 1 = 0 adalah positif. Tentukan a, b, c, d. 10. Untuk bilangan real a, b, dan c yang memenuhi a ≥ b ≥ c > 0, buktikan bahwa a2 − b2 c2 − b2 a2 − c2 + + ≥ 3a − 4b + c. c a b 11. Buktikan bahwa

1 1 3 5 7 99 1 < . . . ··· < . 15 2 4 6 8 100 10

12. Misalkan a1 , a2 , . . . , an bilangan real positif dan b1 , b2 , . . . , bn adalah penataan kembali dari a1 , a2 , . . . , an . Buktikan bahwa an a1 a2 + + ··· + ≥ n. b1 b2 bn 13. Untuk bilangan asli n sebarang, buktikan bahwa n+1  n  1 . (a) 1 + n1 < 1 + n+1 n+2 n+1   1 > 1 + n+1 . (b) 1 + n1 14. Buktikan bahwa √ 15. Buktikan bahwa

1 1 1 √ +√ √ + ··· + √ √ > 24. 1+ 3 5+ 7 9997 + 9999

√ √ √ n n + n n + n n − n n < 2 n n untuk n ≥ 2.

16. Misalkan x1 + x2 + x3 = π2 , xi > 0. Buktikan bahwa 1 sin x1 sin x2 sin x3 ≤ . 8 √ 17. Buktikan bahwa tan a + tan b ≥ 2 tan ab untuk setiap a, b ∈ [0, π2 ). 18. Buktikan bahwa untuk bilangan asli n berlaku   n+1 n . n! ≤ 2 19. Misalkan a1 , a2 , . . . , an dan b1 , b2 , . . . , bn adalah bilangan real positif yang memenuhi a1 a2 . . . an = b1 b2 . . . bn . Buktikan bahwa (a1 b1 + 1) (a2 b2 + 1) . . . (an bn + 1) ≥ 2n . b1 b2 . . . bn

12

20. Cari semua solusi real dari sistem persamaan berikut. x3 + y = 3x + 4 2y 3 + z = 6y + 6 3z 3 + x = 9z + 8 21. Cari semua tripel (x, y, z) yang memenuhi x4 + y 4 + z 4 − 4xyz = −1. 22. Cari semua solusi real dari sistem persamaan √

4z − 1 √ 4x − 1 y+z = z+x = 4y − 1

x+y =

23. Buktikan tidak ada belangan real x, y, x yang memenuhi x2 + 4yz + 2z = 0 x + 2xy + 2z 2 = 0 2xz + y2 + y + 1 = 0 24. Cari semua bilangan real m sehingga persamaan (x2 − 2mx − 4(m2 + 1))(x2 − 4x − 2m(m2 + 1)) = 0 mempunyai tepat tiga akar.

Rujukan [1] Engel, A. 1998. Problem-Solving Strategies. New York: Springer-Verlag. [2] Larson, L. C. 1983. Problem-Solving Through Problems. New York: Springer-Verlag.

13

1.2.1

Relasi dua himpunan

1. Himpunan A dikatakan himpunan bagian dari B kita tulis dengan A ⊆ B jika untuk setiap x ∈ A maka x ∈ B. 2. Himpunan A dikatakan sama dengan himpunan B kita tulis dengan A = B jika A ⊆ B dan B ⊆ A. 3. Himpunan B dikatakan komplemen dari himpunan A kita tulis dengan B = Ac atau B = A jika himpunan B berisi semua anggota dari himpunan semesta yang bukan anggota himpunan A. 1.2.2

Operasi dua himpunan

1. Irisan dua himpunan A dan B dinotasikan dengan A ∩ B adalah himpunan yang anggotaangotanya merupakan anggota dari kedua himpunan A dan B. Secara matematika dapat kita tuliskan A ∩ B = {x|x ∈ A dan x ∈ B} 2. Gabungan dua himpunan A dan B dinotasikan dengan A∪B adalah himpunan yang anggotaangotanya merupakan anggota himpunan A atau himpunan B. Secara matematika dapat kita tuliskan A ∪ B = {x|x ∈ A atau x ∈ B} 3. Selisih dua himpunan A dan B dinotasikan dengan A − B adalah himpunan yang anggotaanggotanya merupakan anggota himpunan A tetapi bukan anggota himpunan B. Secara metematika dapat kita tuliskan A − B = {x|x ∈ A dan x ∈ / B} Definisi 1 (sifat tertutup) Himpunan A dikatakan tertutup terhadap operasi ∗ (bisa penjumlahan, pengurangan, pembagian, perkalian, dan lain-lain) jika untuk setiap a, b ∈ A berlaku a ∗ b ∈ A. Contoh 1. Himpunan bilangan bulat tertutup terhadap penjumlahan, pengurangan, dan perkalian, tetapi tidak terhadap pembagian karena 12 ∈ / Z.

1.3

Himpunan bilangan bulat

Tentu saja himpunan ini telah kita kenal dengan akrab sejak di sekolah dasar. Di sini kita akan membahas sifat-sifat yang berkaitan dengan himpunan bilangan bulat dan himpunan bagiannya. Salah satu himpunan bagian dari himpunan bulat adalah himpunan bilangan asli, himpunan ini beranggotakan bilangan-bilangan bulat yang positif. Beberapa sifat yang berkaitan dengan bilangan bulat dan himpunan bagiannya 1. Himpunan bilangan bulat tertutup terhadap penjumlahan, pengurangan, dan perkalian, 2. Himpunan bilangan asli tertutup tertutup terhdap penjumlahan dan perkalian,

15

3. Setiap himpunan bagian dari himpunan bilangan asli selalu mempunyai elemen terkecil (minimal), 4. Setiap himpunan bagian dari himpunan bilangan bulat yang berhingga selalu mempunyai elemen minimal dan elemen maksimal, Sifat (3) disebut sifat terurut rapi (well ordering principle).

1.4

Soal-soal Latihan

1. Tunjukkan hukum D’Morgan yaitu (A ∩ B)c = Ac ∪ B c dan (A ∪ B)c = Ac ∩ B c . 2. Misalkan A ⊆ Z tertutup terhadap pengurangan. Jika diketahui 4 dan 7 merupakan anggota A, (a) tunjukkan bahwa 0, 100, 208 ∈ A, (b) daftarlah semua anggota dari A. 3. Misalkan S adalah himpunan yang memuat semua bilangan bulat. Jika untuk setiap s ∈ S yang tidak nol, terdapat s ∈ S sehingga ss = 1, maka tentukan semua anggota-anggota S. 4. Jumlah dari rata-rata aritmatik himpunan A dan rata-rata aritmatik himpunan B adalah 5002. Himpunan A dan B terdiri dari bilangan-bilangan asli berurutan. Jika A ∩ B = {2005} tentukan kemungkinan unsur himpunan B yang terbesar. (soal olimpiade matematika tk propinsi tahun 2005) 5. Misalkan S adalah himpunan yang memuat bilangan 1, 2, 3, dan 4. Diketahui untuk sebarang a, b, c, d ∈ S yang semuanya berbeda akan berlaku ab + cd ∈ S. Selidiki apakah 2008 ∈ S? 6. Buktikan sifat well ordering principle pada sebarang sub himpunan bilangan asli. 7. Suatu barisan bilangan bulat {an } memenuhi persamaan aan +n = an untuk setiap bilangan asli n. Jika diketahui a2008 = 1, maka tunjukkan bahwa an = 1 untuk setiap bilangan asli n.

2

Teorema Keterbagian

Pada bab ini, kita akan mempelajari tentang konsep dasar keterbagian, algoritma pembagian, faktor persekutuan terbesar, dan kelipatan persekutuan terkecil.

2.1

Keterbagian

Definisi 1 Misalkan a dan b adalah bilangan bulat dengan a = 0. Bilangan a dikatakan habis membagi b jika terdapat bilangan bulat k sehingga b = ka. Untuk selanjutnya kita tulis a|b, sedangkan dalam hal a tidak habis membagi b kita tulis dengan a  b. Contoh 1 3|12 karena terdapat bilangan bulat k yaitu k = 4 sehingga 12 = 4 × 3 3  7 karena kita tidak mungkin mendapatkan bilangan bulat k sehinga 7 = k × 3. Dari definisi di atas kita dapat menurunkan sefat-sifat sebagai berikut 16

Sifat 1 Untuk setiap bilangan bulat a yang tidak nol selalu berlaku a|a dan a|0 Sifat 2 Untuk setiap bilangan bulat a selalu berlaku 1|a Sifat 3 Jika a|b maka (i) |a| ≤ |b| (ii) ac|bc untuk setiap bilangan bulat c yang tidak nol. Sifat 4 Jika a|b dan a|c maka a| (mb + nc) untuk setiap bilangan bulat m dan n. Di sini, kita hanya akan membuktikan Sifat 4. Sedangkan untuk sifat-sifat yang lainnya diserahkan kepada pembaca sebagai latihan. Bukti Sifat 4 Perhatikan bahwa a|b artinya terdapat bilangan bulat k sehingga b = ka, dan juga kita tahu a|c yang berarti terdapat bilangan bulat l sehingga c = la. Dari kedua fakta tersebut kita punya mb + nc = mka + nla = (mk + nl) a yang berarti a| (mb + nc)  Contoh 2 Tentukan semua bilangan asli n sehingga 3n+25 2n−5 juga merupakan bilangan asli (soal OSN tk Propinsi, 2002) Jawab: Agar 3n+25 2n−5 merupakan bilangan bulat haruslah 2n − 5|3n + 25, di lain pihak kita juga punya 2n − 5|2n − 5. Dengan demikian 2n − 5| (2 (3n + 25) − 3 (2n − 5)) atau ekivalen dengan 2n − 5|65. Dari sini kita simpulkan 2n−5 = 1, 5, 13, atau 65. Yang selanjutnya kita dapatkan solusi n = 3, 5, 9, atau 35.

2.2

Algoritma Pembagian

Teorema 1 Jika a dan b adalah bilangan bulat dan b > 0, maka terdapat dengan tunggal bilangan bulat q dan r sehingga a = bq + r dengan 0 ≤ r < b. Bukti: Padang himpunan ..., a − 3b, a − 2b, a − b, a, a + b, a + 2b, a + 3b, ... Jika barisan tersebut memuat unsur nol, maka terdapat bilangan bulat q sehingga a = bq + r dengan r = 0. Jika barisan tersebut tidak terdapat unsur nol, maka a tidak mungkin nol. Jika a > 0 maka a + ab = a (1 + b) > 0, dan jika a < 0 maka a − ab = −a (b − 1) > 0. Jadi, barisan tersebut memuat unsur positif. Dengan demikian, jika kita himpun semua elemen yang positif sebut saja himpunan S, maka menurut well ordering principle S mempunyai elemen terkecil, sebut elemen minmal tersebut adalah r = a − qb. Kita akan buktikan bahwa r < b. Jelas r = b (mengapa?), andaikan r > b maka akan kita peroleh

17

s = a−(q + 1) b = a−qb−b = r −b > 0. Perhatikan bahwa s ∈ S, dan s < r. Ini kontradiksi dengan asumsi bahwa r merupakan elemen terkecil. Jadi haruslah terdapat bilangan bulat q sehingga 0 < r = a − bq < b atau dengan kata lain a = bq + r dengan 0 < r < b Sekarang akan kita buktikan ketunggalannya. Misalkan terdapat bilangan bulat 0 ≤ r1 , r2 < b dan q1 serta q2 sehingga a = bq1 + r1 = bq2 + r2 . Dari sini akan diperoleh b (q1 − q2 ) = r2 − r1 yang berarti b| (r2 − r1 ) , akan tetapi −b < r2 − r1 < b , akibatnya r2 − r1 = 0 atau dengan kata lain r2 = r1 . Dengan fakta r2 = r2 ini juga akan berakibat q1 = q2 dan kita selesai. Dari teorema di atas, dapat kita pahami bahwa jika m suatu bilangan asli, maka untuk sebarang bilangan bulat n dapat dinyatakan sebagai n = mk + r untuk suatu bilangan bulat k dan r dengan 0 ≤ r ≤ m − 1. Bilangan yang berbentuk mk + r adalah bilangan bulat yang bersisa r ketika dibagi m. Sebagai contoh, kita kita ambil m = 2, maka fakta di atas mengatakan bahwa setiap bilangan bulat dapat dinyatkan dalam bentuk 2k atau 2k + 1, yang selanjutnya dalam kehidupan kita sehari-hari bilangan yang berbentuk 2k dan 2k + 1 berturut-turut kita katakan bilangan genap dan bilangan ganjil. Sekarang, marilah kita lihat beberapa contoh berikut: Contoh 3 Tentukan semua banyak bilangan asli n dengan n < 2008 yang menyebabkan 13 n (n + 1) merupakan bilangan bulat. Jawab: Setiap bilangan asli dapat dinyatakan dalam bentuk 3k, 3k + 1, atau 3k + 2 Untuk n = 3k, kita punya 13 .3k. (3k + 1) = k (3k + 1) merupakan bilangan bulat, untuk n = 3k + 1 kita punya 13 (3k + 1) (3k + 2) = 3k 2 + 3k + 23 bukan merupakan bilangan bulat, untuk n = 3k + 2 kita punya 13 (3k + 2) (3k + 3) = (3k + 2) (k + 1) merupakan bilangan bulat. Jadi, bilangan asli n yang menyebabkan 13 n (n + 1) bukan bilangan bulat adalah bilangan asli yang berbentuk 3k + 1. Bilangan seperti ini yang kurang dari 2008 dapat kita daftar dengan cara berikut: 1 = 3.0 + 1, 4 = 3.1 + 1, 7, ..., 2005 = 3.668 + 1 yang berarti ada 669 bilangan asli kurang dari 2008 yang berbentuk 3k + 1. Dengan ddemikian, banyak bilangan asli n < 2008 yang menyebabkan 13 n (n + 1) merupakan bilangan bulat adalah 2007 − 669 = 1338. Contoh 4. Tunjukkan bahwa tidak ada bilangan kuadrat pada barisan 11, 111, 1111, 11111, ... Jawab: 2 2 Perhatikan  2 bahwa untuk n = 2k kita punya n = 4k dan untuk n = 2k + 1 kita punya 2 n = 4 k + k + 1. Dari sini kita dapat simpulkan bahwa sisa pembagian dari bilangan kadrat oleh 4 adalah 0 atau 1. Sekarang perhatikan barisan 11, 111, 1111, 11111, ... 18

sisa pembagian setiap suku oleh 4 selalu bersisa 3, dengan demikian tidak ada bilangan kuadrat pada barisan di atas.

2.3 2.3.1

Pembagi sekutu terbesar dan faktor sekutu terkecil Pembagi sekutu terbesar

Pada saat sekolah dasar, kita semua tentu telah mengenal pembagi sekutu terbesar atau biasa disebut faktor persekutuan terbesar (FPB), atau disebut juga greatest common divisor (gcd). Definisi 2. Diberikan a dan b adalah bilangan bulat yang tidak keduanya nol. Bilangan asli d disebut pembagi sekutu terbesar dari a dan b atau ditulis dengan d = gcd (a, b) jika (i). d|a dan d|b (ii). untuk setiap bilangan asli c dengan c|a dan c|b haruslah berlaku c ≤ d. bagian (i) mengatakan bahwa d adalah pembagi sekutu dari a dan b, sedangkan bagian (ii) mengatakan bahwa untuk setiap pembagi sekutu dari a dan b harus lebih kecil atau sama dengan d, dengan kata lain (ii) mengatakan bahwa d merupakan pembagi sekutu yang terbesar. Definisi 3. Bilangan bulat a dan b dikatakan saling prima (relatif prima) jika gcd (a, b) = 1. Definisi 4. Untuk sebarang bilangan bulat a, b dan c didefinisikan gcd (a, b, c) = gcd (gcd (a, b) , c) = gcd (a, gcd (b, c)) Dari definisi di atas, dapat diturunkan beberapa sifat di bawah ini: 1. gcd (a, b) = gcd (b, a) = gcd (|a|, |b|) 2. gcd (a, 1) = 1 untuk setiap bilangan bulat a, 3. gcd (a, 0) = |a| untuk setiap bilangan bulat tak nol a, 4. gcd (ma, mb) = |m| gcd (a, b) untuk setiap bilangan bulat tak nol m,   5. jika d = gcd (a, b) maka gcd ad , db = 1. Bukti untuk sifat (5) dapat dilihat pada contoh 6, sedangkan untuk yang lain diserahkan kepada pembaca sebagi latihan.

19

2.3.2

Menentukan gcd dua bilangan dengan algoritma Euclide

Misalkan a dan b bilangan bulat yang tidak keduanya nol. Kita akan menghitung gcd dari a dan b dengan menggunakan algoritma pembagian yang telah kita kenal pada sub bab sebelumnya. Karena gcd (a, b) = gcd (b, a) = gcd (|a|, |b|) , maka di sini hanya akan dibahas untuk a dan b bilangan asli dengan a > b. Berdasarkan algoritma pembagian, akan terdapat bilangan bulat q dan r dengan 0 ≤ r < b sehingga a = bq + r atau ekivalen dengan r = a − bq. Perhatikan bahwa untuk setiap pembagi sekutu a dan b pasti merupakan pembagi dari r. Oleh sebab itu, dapat kita simpulkan gcd (a, b) = gcd (b, r) . Jika r = 0, maka gcd (a, b) = gcd (b, 0) = b. Jika r = 0 kita dapat lakukan langkah yang sama pada b dan r, yakni terdapat q1 dan r1 dengan 0 ≤ r1 < r sehingga b = rq1 + r1 . Dengan argumen yang sama seperti sebelumnya, kita simpulkan gcd (b, r) = gcd (r, r1 ) . Jika r1 = 0, maka gcd (b, r) = gcd (r, r1 ) = r. Jika tidak, kita dapat melakukan langkah di atas sehingga kita peroleh barisan r1 , r2 , ... Akan tetapi, karena a dan b berhingga, maka tentu akan terdapat n sehingga rn = 0. Dengan demikian gcd (a, b) = gcd (b, r) = gcd (r, r1 ) = gcd (r1 , r2 ) = ... = gcd (rn−1 , rn ) = gcd (rn−1 , 0) = rn−1 Untuk lebih jelasnya, perhatikan contoh berikut: Contoh 5. Hitung gcd (2008, 123456) . Jawab: gcd (2008, 123456) = gcd (123456, 2008) . Dengan algoritma pembagian 123456 = 61×2008+968, dengan demikian gcd (123456, 2008) = gcd (2008, 968) . Kemudian 2008 = 2 × 968 + 72, sehingga gcd (2008, 968) = gcd (968, 72) , seterusnya dengan algoritma pembagian akan kita peroleh gcd (968, 72) = gcd (72, 32) = gcd (32, 8) = gcd (8, 0) = 8 dengan demikian gcd (2008, 123456) = 8. Contoh 6.   Jika d = gcd (a, b) maka tunjukkan bahwa gcd ad , db = 1. Jawab:   Misalkan gcd ad , db = k, maka kita punya k| ad dan k| db , yang berakibat kd|a dan kd|b dan selanjutnya kita peroleh kd ≤ d (mengapa?). Dengan demikian, k ≤ 1 dan kita peroleh k = 1  (karena k merupakan bilangan asli). Jadi gcd ad , db = 1. Teorema 2(Identitas Benzout) Jika d = gcd (a, b) maka terdapat bilangan bulat x dan y sehingga ax + by = d. Jawab: Bentuk himpunan S = {ax + by|x, y ∈ Z, ax + by > 0} perhatikan bahwa jika kita ambil x = a dan n = b kita punya a2 + b2 > 0 (ingat pada pendefinisian gcd kita asumsikan a dan b tidak keduanya nol) yang berarti S tidak kosong. Dengan demikian S mempunyai elemen terkecil, sebut saja d. Kita akan buktikan bahwa d = gcd (a, b) . Pertama akan kita buktikan bahwa d|a dan d|b. Dengan algoritma pembagian kita dapat tulis a = dq + r dengan 0 ≤ r < t atau dengan kata lain r = a − dq. Akan tetapi d ∈ S yang berarti d = am + bn untuk suatu m, n ∈ Z. Oleh karena itu, kita punya r = a − dq = a − (am + bn) q = a − aqm − bnq = a (1 − qm) − b (nq) ∈ S. Karena d adalah elemen terkecil dari S dan r < t maka r = 0 yang berarti

20

d|a, dengan cara yang sama kita peroleh juga d|b. Sekarang misalkan c adalah sebarang bilangan asli dengan c|a dan c|b, maka c|am + bn atau c|d. terbukti bahwa d = gcd (a, b) . Dengan menggunakan teorema di atas kita dapat menurunkan beberapa sifat sebagai berikut: 1. Jika d = gcd (a, b) maka untuk sebarang bilangan bulat c dengan c|a dan c|b haruslah berlaku c|d. 2. Jika a|bc dan gcd (a, b) = 1, maka a|c. Bukti: Untuk sifat (1) dapat langsung dilihat dari pembuktian teorema identitas Benzout, sehingga di sini hanya akan kita buktikan untuk sifat (2) . Bukti sifat (2) : Perhatikan bahwa a|bc, artinya terdapat bilangan bulat k sehingga bc = ka. Selain itu, kita juga punya gcd (a, b) = 1. Menurut identitas Benzout kita dapat menemukan bilangan bulat x dan y dengan sifat ax + by = 1. Dengan mengalikan kedua ruas dengan c akan kita peroleh acx + bcy = c yang ekivalen dengan acx = c − bcy. Perhatikan bahwa a| (c − bcy) , akan tetapi karena a|bcy akan berakibat a|c. Contoh 7. Diberikan gcd (15, 24) = 3. Cari salah satu pasangan bulat (x, y) sehingga 15x + 24y = 3. Jawab: Perhatikan bahwa 24 = 15.1 + 9, 15 = 9.1 + 6, dan 9 = 6.1 + 3 (ingat mencari gcd dengan Algoritma Euclide). Dengan demikian 3 = 9 − 6 = 9 − (15 − 9) = 2.9 − 15 = 2 (24 − 15) − 15 = −3.15 + 2.24 kita dapat mengambil x = −3 dan y = 2. Contoh 8 Jika gcd (a, b) = 1 dan gcd (a, c) = 1, tunjukkan bahwa gcd (a, bc) = 1. Jawab: kita punya gcd (a, b) = 1 dan gcd (a, c) = 1, sehingga ada bilangan bulat x dan y yang memenuhi ax + by = 1 dan ada bilangan bulat m dan n yang memenuhi ax + cy = 1. Kemudian kita peroleh (ax + by) (am + cn) = 1 ⇐⇒ a (axm + bmy + cnx) + bc (ny) = 1 yang berakibat gcd (a, bc) |1 dan tentunya gcd (a, bc) = 1.

Kelipatan persekutuan terkecil. Selain pembagi sekutu terbesar, tentunya pada saat sekolah dasar juga kita telah mengenal kelipatan persekutuan terkecil (KPK). Dalam pembahasan selanjutnya, untuk sebarang bilangan bulat a dan b KPK dari a dan b kita tulis dengan [a, b] . Definisi 5 Diberikan bilangan bulat a dan b yang tidak keduanya nol. Bilangan bulat positif m disebut KPK dari a dan b jika 1. m|a dan m|b, 2. untuk setiap bilangan bulat positif n dengan n|a dan n|b haruslah berlaku m ≤ n.

21

Definisi 6 Misalkan a, b, dan c bilangan bulat yang tidak semuanya nol, KPK dari a, b, dan c didefinisikan sebagai [a, b, c] = [[a, b] , c] = [a, [b, c]] Langsung dari definisi di atas, kita dapat menurunkan beberapa sifat sederhana sebagai berikut:

1. [a, b] = [b, a] untuk setiap bilangan bulat a dan b yang tidak keduanya nol, 2. [a, 0] = 0 untuk setiap bilangan bulat tak nol a, 3. [a, 1] = |a| untuk setiap bilangan bulat a. Teorema 3 Jika a dan b adalah bilangan bulat yang tidak keduanya nol, maka ab gcd (a, b)

[a, b] =

Bukti: Misalkan d = gcd (a, b) , maka a = da1 dan b = db1 untuk suatu bilangan bulat a1 , b1 dengan gcd (a1 , b1 ) = 1. Misalkan m = da1 b1 . Akan kita buktikan bahwa m = [a, b] . Jelas bahwa a|m dan b|m. Ambil sebarang bilangan asli n dengan sifat a|n dan b|n, artinya n = ka dan n = lb untuk suatu bilangan bulat k, l. Dari sini kita dapatkan ka = lb ⇐⇒ kda1 = ldb1 ⇐⇒ ka1 = lb1 . Kita punya a1 |lb1 , dan karena gcd (a1 , b1 ) = 1 maka a1 |l yang berarti l = ta1 untuk suatu bilangan bulat t. Dengan demikian kita punya n = lb = ldb1 = tda1 b1 , akibatnya m|n dan kita selesai membuktikan m = [a, b] . Contoh 9. Hitung [56, 72] . Jawab: Karena gcd (56, 72) = 8 maka [56, 72] =

56.72 8

= 504.

Contoh 10. Tentukan bilangan bulat positif terkecil lebih dari 1 yang bersisa 1 ketika dibagi k untuk setiap 2 ≤ k ≤ 10. Jawab: Bilangan yang bersisa 1 ketika dibagi k pasti berbentuk km + 1 untuk suatu bilangan bulat m, dan karena bilangan tersebut harus berbentuk km + 1 untuk setiap 2 ≤ k ≤ 10 maka bilangan tersebut harus berbentuk rm + 1 dengan r habis dibagi 2 ≤ k ≤ 10. Dengan demikian, r = [2, 3, 4, ..., 10] = 2520. Jadi bilangan yang dimaksud pasti berbentuk 2520m+1 untuk suatu bilangan bulat m. Dan karena kita mencari yang terkecil dan lebih besar dari 1 maka kita ambil m = 1. Jadi, bilangan yang dimaksud adalah 2521.

2.4

Soal-soal Latihan

1. Tentukan semua bilangan bulat p yang menyebabkan (a)

8p+9 2p+1

merupakan bilangan bulat, 22

(b) 2p + 1 membagi 2p2 + 7, (c) p2 − 10 kelipatan p + 10. 2. Tentukan semua bilangan asli n sehingga

n3 +24 n+3

juga merupakan bilangan asli.

3. Tentukan bilangan asli terbesar n sehingga n3 + 100 kelipatan n + 10. 4. Diberikan f (x) = ax2 + bx + c, dengan a, b, dan c adalah bilangan bulat. Jika 3|f (x) untuk setiap bilangan bulat x, tunjukkan bahwa 3|a, 3|b, dan 3|c. 5. Buktikan bahwa

n(n+1)(2n+1) 6

merupakan bilangan bulat untuk sebarang bilangan bulat n.

6. Buktikan pernyataan-pernyataan di bawah ini: (a) hasil kali 2 bilangan bulat yang berurutan selalu habis dibagi 2, (b) hasil kali 3 bilangan bulat yang berurutan selalu habis dibagi 6, (c) hasil kali n bilangan bulat yang berurutan selalu habis dibagi n!. 7. Buktikan bahwa gcd (a, b) = gcd (3a + 5b, 11a + 18b) . 8. Buktikan beberapa pernyataan berikut (a) jika gcd (a, b) dan c|a maka gcd (b, c) = 1 (b) jika gcd (a, b) = 1 dan c| (a + b) maka gcd (a, c) = gcd (b, c) = 1   (c) jika gcd (a, b) = 1 maka gcd a2 , b2 = 1. n

9. Misalkan an = k 2 , dengan k bilangan asli. Tunjukkan bahwa jika m = n maka gcd (am , an ) = 1 untuk k genap, dan gcd (am , an = 2) untuk k ganjil. 10. Jika a, m, n bilangan asli, a > 1 dan gcd (m, n) = d, tunjukkan bahwa [am − 1, an − 1] =

(am − 1) (an − 1) ad − 1

(petunjuk: tunjukkan bahwa gcd (am − 1, an − 1) = ad − 1).

3 3.1

Bilangan Prima Pengertian bilangan prima

Pada pembahasan keterbagian, kita kenal istilah a membagi b. Nah, untuk selanjutnya pernyataan a membagi b dapat kita katakan a faktor dari b. Definisi 1. Bilangan bulat positif p dikatakan bilangan prima jika p mempunyai tepat dua faktor positif yaitu 1 dan p sendiri. Definisi 2. Bilangan bulat positif n dikatakan bilangan komposit jika n mempunyai lebih dari 2 faktor positif. 23

Definisi 2 di atas juga dapat kita katakan bahwa n adalah bilangan komposit jika terdapat bilangan bulat positif a, b > 1 sehingga n = ab. Contoh 1. Bilangan-bilangan 2, 3, 5, 7, ... merupakan bilangan prima. Bilangan 4, 6, 8, ... merupakan bilangan komposit. Teorema 1 Banyak bilangan prima adalah tak hingga. Bukti: Andaikan hanya ada sejumlah berhingga bilangan prima, sebut saja p1 , p2 , ..., pn dengan p1 < p2 < ... < pn . Bentuk N = p1 p2 ...pn + 1, jelas bahwa N > pn . Perhatikan bahwa untuk setiap k = 1, 2, ..., n haruslah pk tidak membagi N, karena jika pk membagi N maka pk |1 yang jelas tidak mungkin. Dengan demikian N prima atau terbagi oleh bilangan prima yang lebih dari pn . Hal ini kontradiksi dengan asumsi kita. Jadi banyak bilangan prima adalah tak hingga. Contoh 2. Tentukan semua bilangan prima yang berbentuk n3 + 1 untuk suatu bilangan asli n. Jawab:   Perhatikan bahwa n3 + 1 = (n + 1) n2 − n + 1 , dengan demikian salah satu faktor yaitu n + 1 atau n2 − n + 1 harus sama dengan 1. Jika n + 1 = 1 maka n = 0 (tidak memenuhi), dan jika n2 − n + 1 = 1 maka n = 0 atau n = 1. Untuk n = 1 kita peroleh n3 + 1 = 2. Jadi bilangan prima yang berbentuk n3 + 1 hanyalah 2. Contoh 3.   Tunjukkan bahwa untuk setiap bilangan prima p selalu berlaku 6| p2 − 1 . Jawab: Setiap bilangan prima lebih p > 3 selalu dapat kita nyatakan sebagai 6k + 1 atau 6k − 1. Akan 2 2 tetapi, apapun yang terjadi akan selalu  2 kita  peroleh p = 6m + 1 di mana m = 6k + 2k atau 2 m = 6k − 2k. Dengan demikian 6| p − 1 . Akibat 1. 1. Jika p prima, maka untuk sebarang bilangan asli n berlaku p|n atau gcd (p, n) = 1. 2. Jika p prima dan p|ab untuk suatu bilangan bulat a dan b, maka p|a atau p|b. Bukti: 1. Jika p|n, maka tidak ada yang perlu dibuktikan. Asumsikan p  n. Misalkan d = gcd (p, n) , yang berarti d|p dan d|n. Karena p prima maka d = 1 atau d = p. Dari asumsi kita punya p  n, akibatnya d = n .Dengan kata lain d = 1 dan kita selesai. 2. Jika p|a, maka kita selesai. Asumsikan p  a. Menurut sifat (1) kita punya gcd (p, a) = 1, dengan demikian terdapat bilangan bulat x dan y sehingga px + ay = 1. Dengan mengalikan kedua ruas dengan b akan kita peroleh pbx + aby = b atau setara dengan p (bx) = b − aby. Akan tetapi p|ab akibatnya p|b. Akibat 2. Jika p prima dan n, m sebarang bilangan asli dengan p|nm maka p|n. Bukti langsung dapat dilihat dari akibat 1 bagian 2.

24

3.2

Faktorisasi Prima

Bilangan prima merupakan bilangan yang lebih sederhana daripada bilangan komposit karena bilangan prima hanya mempunyai 2 faktor positif yang berbeda. Oleh karena itu setiap bilangan asli akan kita bawa ke dalam perkalian bilangan-bilangan prima berpangkat yang disebut dengan faktorisasi prima. Berikut penjelasannya. Teorema 2 Setiap bilangan asli n > 1 dapat dinyatakan secara tunggal sebagai n = pa11 pa22 ...pakk dengan k suatu bilangan asli, p1 < p2 < ... < pk bilangan-bilangan prima berbeda , dan ai ≥ 1 untuk setiap i = 1, 2, 3, ..., k. Bukti: Jika n = p prima, maka kita selesai. Asumsikan n komposit. Misalkan p1 adalah bilangan prima terkecil yang membagi n, maka n = p1 n1 untuk suatu bilangan asli n1 . Jika n1 prima, maka kita selesai. Jika n1 tidak prima kita dapat menemukan bilangan prima terkecil yang membagi n1 . Jika bilangan prima tersebut sama dengan p1 maka n = p21 n2 untuk suatu bilangan asli n2 , jika bilangan prima tersebut tidak sama dengan p1 sebut saja p2 maka = p1 p2 n2 . Demikian seterusnya, sampai kita peroleh nm = 1 (mengapa ini dijamin?). Jadi n = pa11 pa22 ...pakk bm dengan Sekarang akan kita buktikan ketunggalannya. Misalkan n = pa11 pa22 ...pakk = q1b1 q2b2 ...qm p1 < p2 < ... < pk dan q1 < q2 < ... < qm bilangan-bilangan prima. Jika terdapat pi yang tidak bm yang berarti p  n dan ini tidak sama dengan qt untuk setiap t = 1, 2, ..., m maka pi  q1b1 q2b2 ...qm i mungkin. Jadi, untuk setiap pi pasti terdapat t ∈ {1, 2, ..., m} sehingga pi = qt . Dengan cara yang sama dapat dibuktikan juga bahwa untuk setiap qi pasti terdapat t ∈ {1, 2, ..., k} sehingga qi = pt . Dengan demikian kita punya k = m. Dan karena p1 < p2 < ... < pk dan q1 < q2 < ... < qk maka pi = qi untuk setiap i = 1, 2, ..., k atau dengan kata lain

pa11 pa22 ...pakk = pb11 pb22 ...pbkk Jika terdapat i sehingga ai > bi maka dengan membagi masing-masing ruas dengan pbi i akan berakibat pi membagi ruas kiri tetapi tidak membagi ruas kanan yang jelas tidak mungkin. Demikian juga jika terdapat i sehingga ai < bi maka dengan membagi kedua ruas dengan pai i akan berakibat pi tidak membagi ruas kiri tetapi pi membagi ruas kanan yang jelas tidak mungkin. Jadi kita harus punya ai = bi untuk setiap i = 1, 2, ..., k. Dan kita selesai membuktikan ketunggalannya.  keterangan: Tidak hanya bilangan asli saja yang dapat kita tulis dalam bentuk perkalian faktor-faktor prima. Secara umum, untuk sebarang bilangan bulat n = 0 selalu dapat ditulis dalam bentuk n = upa11 pa22 ...pakk dengan u = ±1, k bilangan asli, dan p1 , p2 , ..., pk bilangan prima. Faktorisasi prima ini, akan memudahkan kita dalam menganalisa suatu bilangan bulat. Perhatikan bahwa jika bentuk faktorisasi prima dari n adalah n = pa11 pa22 ...pakk maka kita mengetahui beberapa hal sebagai berikut: 25

1. n mempunyai k faktor prima yaitu p1 , p2 , ..., pk . 2. Banyak faktor positif dari n adalah (1 + a1 ) (1 + a2 ) ... (1 + an ) (mengapa?). Salah satu penggunaan faktorisasi prima adalah mencari FPB dan KPK dua bilangan bulat. Jika kita sudah mendapatkan faktorisasi prima dari dua bilangan bulat, sebut saja a dan b, maka pastilah faktorisasi dari keduanya dapat dinyatakan sebagai a = ps11 ps22 ...pskk dan b = pt11 pt22 ...ptkk dengan pi prima dan si , ti ≥ 0 untuk i = 1, 2, ..., k. Dengan faktorisasi prima ini, dapat dipahami bahwa mk Mk M1 M2 1 m2 gcd (a, b) = pm 1 p2 ...pk dan [a, b] = p1 p2 ...pk di mana mi = min {ai , bi } dan Mi =maks{ai , bi } untuk setiap i = 1, 2, ..., k. Untuk lebih jelasnya, mari kita simak beberapa contoh di bawah ini: Contoh 4 Tentukan FPB dan KPK dari 56 dan 2008. Jawab: Akan kita selesaikan dengan faktorisasi prima, perhatikan 112 = 24 .7 dan 2008 = 23 .251.  4bahwa  3 Dengan demikian gcd (56, 2008) = 2 = 8 dan [112, 208] = 2 (7) (251) = 28 112. Contoh 5 Tentukan jumlahan dari semua faktor positif dari 5.000.000. Jawab: Tentu kita dapat mendaftar semua faktor positif dari 5.000.000 kemudian menjumlahkannya, namun akan butuh waktu yang sangat lama. Nah, sekarang perhatikan bahwa faktorisasi prima a b dari 5.000.000 adalah 26 .57 . Dengan demikian, setiap faktor positifnya berbentuk 2 .5 dengan  a = 0, 1, 2, ..., 6 dan b = 0, 1, 2, ..., 7. Sehingga jumlahan dari semuanya adalah 2a .5b = a,b  7    6   38 −1  a b 7 . 2 3 = 2 −1 2 a=0 b=0 Contoh 6 √ Jika m adalah bilangan asli sehingga m merupakan bilangan rasional, maka tunjukkan bahwa m merupakan kuadrat suatu bilangan asli. Jawab: √ Misalkan m = ab dengan a dan b bilangan asli (mengapa?). Jika b = 1, maka jelas bahwa m = a2 . Asumsikan b > 1, akibatnya a > 1. Kita dapat tulis faktorisasi prima dari a dan b yaitu a = 2 1 −2t1 2s2 −2t2 k −2sk ps11 ps22 ...pskk dan b = pt11 pt22 ...ptkk . Sekarang perhatikan bahwa m = ab2 = p2s p2 ...p2s = 1 k  s1 −t1 s2 −t2 sk −sk 2 p1 p2 ...pk . Karena m bilangan asli dan p1 , p2 , ..., pk bilangan prima maka bilangan yang berada dalam tanda kurung merupakan bilangan asli, dan kita selesai.

3.3

Soal-soal Latihan

1. Tentukan semua bilangan asli n yang menyebabkan n4 + 4 merupakan bilangan prima. 2. Tunjukkan n merupakan bilangan komposit jika dan hanya jika n terbagi oleh bilangan prima √ p dengan p ≤ n. 26

3. Tunjukkan bahwa ada tak hingga bilangan prima yang berbentuk 4n+3 untuk suatu bilangan asli n. 4. Apakah terdapat bilangan asli n sehingga 6n + 5 merupakan jumlahan dari dua bilangan prima? 5. Tentukan semua bilangan prima p sehingga 4p2 + 1 dan 6p2 + 1 juga merupakan bilangan prima. (soal OSN I, Yogyakarta, 2002). 6. Diberikan p > 3 adalah bilangan prima. Jika 1+

1 1 1 a + + ... + = 2 3 p−1 b

dengan a dan b bilangan bulat dan gcd (a, b) = 1, maka tunjukkan bahwa p membagi a. 7. Tunjukkan bahwa jika a, b, c, d ∈ N dan ab = cd, maka bilangan an + bn + cn + dn merupakan bilangan komposit untuk setiap bilangan asli n. 8. Jika faktorisasi prima dari n adalah n = pa11 pa22 ...pakk , maka tunjukkan bahwa m

(a) Hasil kali semua faktor positif dari n adalah n 2 dengan m = (1 + a1 ) (1 + a2 ) ... (1 + ak ) , a1 i  a2 i  ak i  (b) Jumlahan semua faktor positif daro n adalah i=0 p1 i=0 p2 ... i=0 pk . 9. Tunjukkan bahwa n merupakan kuadrat sempurna jika dan hanya jika n mempunyai sejumlah ganjil faktor positif. 10. Terdapat 2008 pintu berjajar diberi nomor 1 sampai 1008 dan semuanya dalam keadaan tertutup. Sekelompok anak, Pi dengan 1 ≤ i ≤ 2008 berjalan melalui jajaran pintu tersebut. Masing-masing anak mengubah kondisi pintu nomor k jka dan hanya jika i membagi k, jika pintunya tertutup diubah menjadi terbuka dan sebaliknya. Cari banyak pintu yang terbuka setelah semua anak melewati jajaran pintu tersebut.

4

Persamaan dan Sistem Persamaan dalam Bilangan Bulat

Dalam menyelesaikan persamaan dan sistem persamaan dalam bilangan bulat (sering juga disebut persamaan Diophantine) tentu akan lebih mudah, karena kita hanya dibatasi penyelesaian dalam bilangan bulat. Sebagai contoh jika kita akan mencari pasangan bilangan real (x, y) yang  memenuhi  xy = 2, tentu akan ada tak hingga banyaknya yaitu semua pasangan bilangan real x, x2 untuk setiap bilangan real tak nol x, pasti merupakan solusi xy = 2. Akan tetapi jika kita akan mencari pasangan bilangan bulat (x, y) yang memenuhi persamaan xy = 2, maka solusinya hanya ada 4 yaitu (1, 2) , (2, 1) , (−1, −2) , dan (−2, −1) . Mengapa demikian? Untuk lebih jelasnya simak uraian berikut:

27

4.1

Persamaan Diophantine Linear

Persamaan ini adalah persamaan yang paling sederhana, karena kita bisa langsung mencari solusi umumnya. Definisi 1 Misalkan a, b, dan c adalah bilangan-bilangan bulat. Persamaan Diophantine berbentuk ax + by = c disebut Persamaan Diophantine linear dan setiap pasangan bilangan bulat (x, y) yang memenuhi ax + by = c disebut solusi. Teorema 1 Persamaan Diophantine ax + by = c mempunyai solusi jika dan hanya jika gcd (a, b) |c. Bukti: =⇒) Diketahui persamaan ax + by = c mempunyai solusi, artinya ada bilangan bulat x0 dan y0 yang memenuhi ax0 + by0 = c. Andaikan gcd (a, b) tidak membagi c. Perhatikan bahwa ruas kiri terbagi oleh gcd (a, b) tetapi ruas kanan tidak terbagi oleh gcd (a, b) yang jelas ini tidak mungkin. Jadi haruslan gcd (a, b) membagi c. ⇐=) Diketahui gcd (a, b) |c, artinya terdapat bilangan bulat k sehingga c = k gcd (a, b) . Menurut identitas Benzout terdapat bilangan bulat m dan n yang memenuhi am + bn = gcd (a, b) . Dengan mengambil x = km dan y = kn kita akan punya ax + by = akm + bkn = k (am + bn) = k (gcd (a, b)) = c yang berarti persamaan ax + by = c mempunyai solusi yaitu (km, kn) . Contoh 2 Hitung banyak bilangan bulat 1 ≤ n ≤ 100 yang dapat dinyatakan dalam bentuka 6x + 8y untuk suatu bilangan bulat x dan y. Jawab: Perhatikan bahwa gcd (6, 8) = 2. Oleh karena itu menurut teorema di atas, hanya bilangan yang terbagi oleh 2 yang dapat dinyatakan dalam bentuk 6x + 8y untuk suatu bilangan bulat x dan y. Dalam hal ini, 1 ≤ n ≤ 100 yang terbagi oleh 2 ada tepat 50 bilangan. Teorema 2 Jika Persamaan Diophantine ax + by = c mempunyai solusi (x0 , y0 ) maka persamaan tersebut mempunyai tak hinga banyaknya solusi dan setiap solusinya berbentuk x (k) = x0 + k

b a dan y (k) = y0 − k gcd (a, b) gcd (a, b)

untuk sebarang bilangan bulat k. Bukti: Diketahui (x0 , y0 ) solusi dari ax + by = c, artinya ax0 + by0 = c. Jika (x (k) , y (k)) kita substitusikan ke persamaan akan kita peroleh ax (k) + by (k) = ax0 + k

ab ab + by0 − k = ax0 + by0 = c gcd (a, b) gcd (a, b)

28

yang berarti (x (k) , y (k)) juga merupakan solusi.Nah, untuk bukti bahwa solusi persamaan ax + by = c hanyalah (x (k) , y (k)) akan kita bahas setelah kita membahas kongruensi bilangan bulat (pada modul ini tidak dibahas). Pertanyaannya adalah bagaimana cara kita menentukan solusi awal (x0 , y0 ) ini? Ingat kembali waktu kita membahas Algoritma Euclide. Kita bisa mencari bilangan bulat m dan n sehingga am + bn = gcd (a, b) . Karena persamaan ax + by = c punya solusi jika dan hanya jika gcd (a, b) |c maka terdapat k sehingga c = k gcd (a, b) . Dengan demikian kita bisa mengambil solusi awal x0 = km dan y0 = kn. (Pada umumnya, bukan pekerjaan yang sulit untuk mencari salah satu solusi dari persamaan ax + by = c). Contoh 3 Tentukan semua solusi dari Persamaan Diophantine linear 6x + 8y = 12. Jawab: Kita punya gcd (6, 8) = 2 dan 2|12, yang berarti persamaan ini punya solusi. Mudah dipahami bahwa salah satu solusinya adalah (2, 0) . Dengan demikian solusi umumnya adalah x (k) = 2 + 4k dan y (k) = −3k untuk sebarang bilangan bulat k. Contoh 4 Ada berapa banyak pasangan bilangan asli (x, y) yang memenuhi persamaan 4x + 6y = 48? Jawab: Untuk menyelesaikan soal ini kita selesaikan seperti biasa yaitu kita cari solusi umumnya, selanjutnya kita batasi nilai k agar solusinya merupakan bilangan asli. Salah satu solusi dari persamaan ini adalah (12, 0) , sehingga solusi umumnya adalah x = 12 + 3k dan y = −2k untuk sebarang bilangan bulat k. Sekarang akan kita batasi nilai k sehingga x, y > 0. Dari 12 + 3k > 0 kita peroleh k > −4 dan dari −2k > 0 kita peroleh k < 0, atau dengan kata lain kita peroleh −4 < k < 0. Akan tetapi, karena k bilangan bulat maka −3 ≤ k ≤ −1. Dengan demikian ada tepat 3 pasangan bilangan asli (x, y) yang memenuhi 4x + 6y = 48.

4.2

Persamaan Diophantine Non Linear

Persamaan ini sangat banyak bentuknya, kita tidak mungkin mengkarakteristik satu persatu. Di sini kita hanya memaparkan dengan beberapa teknik melalui contoh-contoh soal: Contoh 5 (teknik pemfaktoran) Tentukan solusi bulat dari persamaan xy = 2x − y. Jawab: Perhatikan bahwa soal di atas ekivalen dengan xy − 2x + y = 0, dengan menambahkan masingmasing ruas dengan −2, akan diperoleh xy − 2x + y − 2 = −2 dan ini dapat difaktorkan menjadi (x + 1) (y − 2) = −2. Karena x dan y bilangan bulat, maka demikian juga dengan x + 1 dan y − 1. Dengan demikian, ada 4 kejadian yang mungkin (i). x + 1 = −1 dan y − 2 = 2. Dari sini diperoleh solusi x = −2, y = 4, (ii). x + 1 = 1 dan y − 2 = −2. Dari sini diperoleh solusi x = 0, y = 0, (iii). x + 1 = 2 dan y − 2 = −1. Dari sini diperoleh solusi x = 1, y = 1, (iv). x + 1 = −2 dan y − 2 = 1. Dari sini diperoleh solusi x = −3, y = 3.

29

Mudah dicek bahwa keempat pasang solusi memenuhi persamaan yang diberikan. Jadi semua solusinya dapat kita nyatakan dalam pasangan (−2, 4) , (0, 0) , (1, 1) , dan (−3, 3) . Cotoh 6 (teknik pembatasan) Tentukan bilangan asli a, b, c sehingga a1 + 1b + 1c = 1. Jawab: Perhatikan bahwa persamaan di atas simetri, artinya jika a kita tukar dengan b dan b kita tukar dengan a persamaan tidak berubah. Sehingga dapat kita asumsikan a ≥ b ≥ c. Akibatnnya 1 = a1 + 1b + 1c ≤ 1c + 1c + 1c = 3c ⇔ c ≤ 3. Dari sini kita hanya cukup mengecek untuk c = 1, 2, 3. • c = 1, kita substitusikan ke persamaan awal akan kita peroleh penyelesaian bilangan asli.

1 a

1 a

• c = 2, kita substitusikan ke persamaan awal akan kita peroleh Karena a ≥ b, maka kita peroleh .

1 2

=

1 a

+

1 b



2 b

+ 1b = 0, dan ini tidak punya +

1 b

=

1 2

⇔ b ≤ 4. dan juga kita punya b ≥ c = 2.

• b = 2 tidak ada a yang memenuhi. . b = 3, kita peroleh a = 6. . b = 4, kita peroleh a = 4.

• c = 3, kita substitusikan ke persamaan awal akan kita peroleh

1 a

+

1 b

= 23 .

Karena a ≥ b, maka kita peroleh 23 = a1 + 1b ≤ 2b ⇔ b ≤ 3. dan juga kita punya b ≥ c = 3 jadi b = 3, sehingga kita peroleh a = 3. Kita peroleh pasangan solusi (6, 3, 2) , (4, 4, 2) , (3, 3, 3). Perhatikan bahwa awalnya kita asumsikan a ≥ b ≥ c, padahal bisa saja a ≥ c ≥ b atau yang lainnya. Tetapi karena persamaannya simetris, maka solusi yang lainnya tingal diubah urutannya. Jadi semua solusinya adalah (6, 3, 2) , (6, 2, 3) , (3, 2, 6) , (3, 6, 2) , (2, 3, 6) , (2, 6, 3) , (4, 4, 2) , (4, 2, 4) , (2, 4, 4) , dan (3, 3, 3) . Contoh 7 (teknik keterbagian) Tunjukkan bahwa tidak ada bilangan bulat x dan y yang memenuhi pesamaan (x + 1)2 + (x + 2)2 + ... + (x + 99)2 = y 2 Jawab: Andaikan terdapat bilangan bulat x dan y yang demikian. Kita jabarkan yang ruas kiri, yakni:   99.100.199 (x + 1)2 + (x + 2)2 + ... + (x + 99)2 = 99x2 + 99 (100) x + = 99 x2 + 100x + 33.50.199 6  2  2 dengan demikian kita punya 99 x + 100x + 33.50.199 = y . Perhatikan bahwa ruas kiri habis dibagi 3, akibatnya ruas kanan juga habis dibagi 3. Akan tetapi, karena ruas kanan merupakan kuadrat sempurna maka ruas kanan juga kan dibagi 9. Tentu saja ruas kiri juga havis dibagi 9, akibatnya 33.50.199 habis dibagi 9 yang jelas ini tidak mungkin. Contoh 8 (teknik parameter) Tunjukkan bahwa persamaan x2 + y 2 = x3 mempunyai tak hingga banyaknya solusi asli. Jawab: Persamaan di atas dapat kita tulis sebagai y 2 = x3 − x2 atau ekivalen dengan y 2 = x2 (x − 1) . Oleh karena itu, agar persamaan tersebut punya solusi,  2 kitaharus punya x − 1 merupakan kuadrat sempurna. Dengan mengambil x = n2 + 1 dan y = n + 1 untuk sebarang bilangan asli n, maka  2 n  2 mudah ditunjukkan bahwa pasangan n + 1, n n + 1 merupakan solusi yang banyaknya jelas ada tak hingga.

30

4.3

Sistem Persamaan Diophantine

Telah kita bahas beberapa jenis dan contoh Persamaan Diophantine serta cara menyelesaikannya. Nah, untuk menyelesaikan sistem persamaan Diophantine kita dapat membawa ke dalam bentuk Persamaan Diophantine seperti yang telah kita kenal. Pada dasarnya, kita akan mencari solusi bilangan asli yang memenuhi semua persamaan yang diberikan secara simultan.

4.4

Soal-soal Latihan

1. Ada berapa banyak pasangan bilangan asli (x, y) yang memenuhi persamaan 2x+3y = 1000? 2. Banyak pasangan bilangan asli (x, y) yang memnuhi persamaan

1 x



1 y

=

1 3

adalah...

3. Banyaknya bilangan asli n sehingga 3n + 81 merupakan kuadrat sempurna adalah.. 4. Bilangan bulat positif terkecil n sehingga 31 membagi 5n + n adalah... 5. Tentukan semua bilangan asli a, b, dan c yang menyebabkan asli.

1 a

+

1 b

+

1 c

merupakan bilangan

6. Tentukan semua pasangan bilangan bulat non negatif yang memenuhi persamaan (xy − 7)2 = x2 + y 2 7. Diketahui x, y, z, dan n adalah bilangan-bilangan asli yang memenuhi xn + y n = z n Tunjukkan bahwa x, y, dan z semuanya lebih besar dari n. 8. Tentukan semua bilangan real a sehingga persamaan kuadrat x2 + ax + 6a = 0 mempunyai dua solusi yang keduanya bulat. 9. Tentukan semua pasangan bilangan bulat (x, y) yang memenuhi x4 + x3 + x2 + x + 1 = y 2 10. Carilah semua bilangan prima p sehingga sistem persamaan  p + 1 = 2x2 p2 + 1 = 2y 2 mempunyai solusi bulat.

5

Fungsi Tangga

Dalam dunia jual beli, biasanya penjual ingin menjual harganya semahal mungkin, dan sebaliknya pembeli ingin membeli barang yang ia inginkan semurah mungkin. Bahkah, kadang-kadang jika harganya tidak bulat ribuan misalnya 7.300 maka penjual ingin dibayar 7.500, sebaliknya pembeli ingin membayar dengan harga yang dibulatkan ke bawah yaitu 7.000. Nah, dalam ilmu matematika kita akan mengenal fungsi yang digunakan oleh penjual dan pembeli di atas. Fungsi yang akan kita pelajari jika digambarkan pada bidang kartesius akan berbentuk seperti tangga. Ada 3 macam fungsi tangga yang akan kita bahas dalam bab ini, yaitu fungsi floor (pembulatan ke bawah), fungsi ceiling (pembulatan ke atas), dan fungsi bulat (pembulatan ke bilangan bulat yang terdekat).

31

5.1

Fungsi floor

Fungsi floor disebut juga fungsi pembulatan ke bawah, yakni dengan mengambil bagian bulatnya. Untuk sebarang bilangan real x,nilai fungsi floor dari x kita tulis dengan x . Definisi 1 Misalkan x adalah sebarang bilangan real. Nilai fungsi floor x kita tulis dengan x merupakan bilangan bulat terbesar yang kurang dari atau sama dengan x. Contoh: √

3, 14 = 3, −2, 5 = −3, 2 = 1, dan lain sebagainya. Definisi 2 Untuk sebarang bilangan real x, notasi {x} menyatakan bagian pecahan dari x. Secara matematika, definisi di atas dapat kita tuliskan {x} = x − x Dari sini jelas bahwa untuk sebarang bilangan real x berlaku 0 ≤ {x} < 1. Contoh: √ 2 = 0, 41..., dan lain sebagainya. {3, 14} = 0, 14; {−2, 5} = 0, 5; Langsung dari definisi, kita dapat menurunkan beberapa sifat sebagai berikut: 1. Untuk sebarang bilangan real x selalu berlaku x − 1 < x ≤ x, 2. x = x jika dan hanya jika x ∈ Z, 3. x + k = x + k untuk sebarang bilangan bulat k, 4. x + y ≤ x + y untuk setiap x, y ∈ R, 5. xy ≤ x y untuk setiap x, y ∈ R. Sifat 1, 2, dan 3 trivial. Di sini kita hanya akan membuktikan sifat 4, sedangkan untuk sifat 5 buktinya hampir sama dengan pembuktian sifat 4 dan diserahkan kepada pembaca sebagai latihan. Bukti sifat 4. Tulis x = x + {x} dan y = y + {y} , akan kita peroleh x + y = x + y + {x} + {y} = x + y + {x} + {y} ≥ x + y dan kita selesai. 

1 2

Contoh 1

Untuk sebarang bilangan real x, tunjukkan bahwa x + 12 + x = 2x . Bukti: Tulis x = x + {x} dengan 0 ≤ {x} < 1. Kita bagi 2 kasus, yaitu jika 0 ≤ {x} < ≤ {x} < 1.

(i). untuk 0 ≤ {x} < 12 , kita punya 





1 1 1 + x = x + {x} + + x = 2 x + {x} + = 2 x x+ 2 2 2 dan 2x = 2 x + 2 {x} = 2 x + 2 {x} = 2 x yang jelas bahwa ruas kiri sama dengan ruas kanan. 32

1 2

dan

(ii). untuk

1 2

≤ {x} < 1, kita punya 





1 1 1 + x = x + {x} + + x = 2 x + {x} + = 2 x + 1 x+ 2 2 2

dan 2x = 2 x + 2 {x} = 2 x + 2 {x} = 2 x + 1 yang juga jelas bahwa ruas kiri sama dengan ruas kanan. Perhatikan bahwa jika n dan a adalah sebarang bilangan asli, mudah dipahami bahwa n/a merupakan banyaknya bilangan kelipatan a di antara 1, 2, ..., n. Fakta ini sederhana tetapi dapat kita gunakan untuk menyelesaikan beberapa permasalahan seperti pada contoh berikut. Contoh 2 Tentukan banyak anggota himpunan {1, 2, ..., 100} yang habis dibagi 2 atau 3. Jawab: Banyak bilangan kelipatan 2 ada 100/2 = 50, banyak bilangan kelipatan 3 ada 100/3 = 33, dan banyak bilangan kelipatan 2 dan 3 ada 100/6 = 16. Oleh karena itu, dengan prinsip inklusi eksklusi kita peroleh bahwa banyak kelipatan 2 atau 3 ada 100/2 + 100/3 − 100/6 = 50 + 33 − 16 = 67 bilangan. Contoh 3 Hitung banyak nol di sebelah kanan tanpa terputus dari 31!. Jawab: Angka nol di sebelah kanan tanpa terputus pada 31! akan dihasilkan pada saat kita mengalikankelipatan 10, dan faktor 10 ini didapat dari kelipatan genap dan kelipatan 5. Karena banyak kelipatan 2 lebih banyak daripada banyak kelipatan 5, maka kita cukup menghitung kelipatan 5. Perhatikan juga bahwa mungkin bilangan kelipatan 5 yang kita kalikan tersebut juga merupakan kelipatan 25 dan jika dikalikan dengan bilangan kelipatan 4 akan menghasilkan bilangan kelipatan 100 yang akan menambah nol di sebelah kanan sebanyak 2, akan tetapi sebanyak 1 nol telah telah kita masukkan saat kita menghitung kelipatan 5. Dengan demikian, banyak nol di sebelah kanan tanpa terputus dari 31! adalah 31/5 + 31/25 = 6 + 1 = 7 nol.

5.2

Fungsi ceiling

Fungsi floor disebut juga fungsi pembulatan ke atas. Untuk sebarang bilangan real x,nilai fungsi ceiling dari x kita tulis dengan x . Definisi 3 Misalkan x adalah sebarang bilangan real. Nilai fungsi ceiling x kita tulis dengan x merupakan bilangan bulat terkecil yang lebih dari atau sama dengan x. Contoh: √  3, 14 = 4, −2, 5 = −2, 2 = 2, dan lain sebagainya.

5.3

Fungsi bulat

Fungsibulat disebut juga fungsi pembulatan ke bilangan bulat terdekat. Untuk sebarang bilangan real x,nilai fungsi bulat dari x kita tulis dengan [x] . Definisi 4 33

Misalkan x adalah sebarang bilangan real. Nilai fungsi bulat x kita tulis dengan [x] merupakan bilangan bulat terdekat dengan x. Jika x = k+ 12 untuk suatu bilangan bulat k, maka kita definisikan [x] = k + 1. Contoh: √  [3, 14] = 3, [−2, 5] = −2, 2 = 1, dan lain sebagainya Dari definisi di atas, kita dapat menurunkan beberapa sifat berikut: 1. x ≤ [x] ≤ x untuk setiap bilangan real x. 2. x = x jika dan hanya jika [x] = x, 3. x + k = x + k dan [x + k] = [x] + k untuk sebarang bilangan bulat k, 4. x + y ≥ x + y untuk setiap x, y ∈ R, 5. xy ≤ x y untuk setiap x, y ∈ R. Contoh 4

Tunjukkan bahwa untuk setiap bilangan real x selalu berlaku x + 12 = [x] Jawab:

Tulis x = x + {x} , dengan 0 ≤ {x} < 1. Jika 0 ≤ {x} < 12 maka x + 12 = x dan [x] = x .

Jika 12 ≤ {x} < 1 maka x + 12 = x + 1 dan [x] = x + 1. Contoh 5 Tentukan semua bilangan real x yang memenuhi persamaan  2 2

x + x = 2003 (soal OSN SMA tahun 2003, bidang matematika) Jawab:  

2003 2 Perhatikan bahwa jika x2 bulat maka x2 = x2 yang akan berakibat  2  x =2 2 yang jelas 2 Akibatnya x = x + 1, sehingga bukan 2 bilangan bulat. Jadi, x harus bukan bilangan 2bulat.

2 x + 1 = 2003 yang selanjutnya kita peroleh x = 1001. Dari sini kita simpulkan 1001 ≤ √ √ 2 < 1002 yang kemudian kita dapatkan penyelesaiannya adalah − 1002 < x ≤ − 1001 atau x √ √ 1001 ≤ x < 1002.

5.4

Soal-soal Latihan

1. Buktikan atau beri contoh penyangkal dari pernyataan-pernyataan berikut (a) [x + y] ≤ [x] + [y] untuk setiap x, y ∈ R, (b) [xy] ≤ [x] [y] untuk setiap x, y ∈ R,  

x (c) x = m untuk setiap x ∈ R dan bilangan bulat m. m 2. Misalkan x dan y adalah bilangan real yang memenuhi x + y = x + y dan −x − y = −x + −y . Buktikan bahwa x atau y merupakan bilangan bulat.

34

3. Untuk setiap bilangan real x didefinisikan x sebagai bilangan bulat terbesar yang kurang dari atau sama dengan x. Misalkan a dan b adalah bilangan real positif sehingga a a = 17 dan b b = 11. Tentukan nilai dari a − b. 4. Hitung 1 1 1 1 + + + ... + [1] [2] [3] [1000] 5. Cari semua bilangan asli n sehingga banyak nol di sebelah kanan tanpa terputus dari n! tepat ada 10 nol. 6. Diketahui S = {1, 2, ..., 100} . Hitung banyaknya anggota S yang merupakan kelipatan 2 atau kelipatan 3 tetapi bukan kelipatan 5. 7. Tentukan semua bilangan real x yang memenuhi persamaan x x = x2 8. Hitung banyak bilangan real x dengan 1 ≤ x ≤ 100 yang memenuhi persamaan

x2 − x2 = (x − x)2     2  2 (m+2)2 + merupakan 9. Tunjukkan bahwa untuk setiap m bilangan asli, m3 + (m+1) 3 3 kuadrat sempurna.  √ n  . Tunjukkan bahwa an ganjil jika n genap, dan genap jika n 10. Misalkan an = 1 + 2 ganjil.

35

1.1

Konkurensi dan Kolinearitas pada Segitiga: Teorema Ceva dan Teorema Menelaos

Kita mulai dengan segmen-segmen garis yang menghubungkan titik sudut suatu segitiga dengan sebuah titik yang terletak pada sisi di depan titik sudut tersebut. Segmen garis seperti itu disebut sebagai cevian (diambil dari nama Giovanni Ceva, seorang matematikawan Italia yang pertama kali menyinggung masalah konkurensi tiga buah cevian). Teorema Ceva. Misalkan ABC sebuah segitiga dan D, E, F tiga titik yang berturut-turut terletak pada sisi-sisi BC, CA, AB. Maka garis-garis AD, BE, CF konkuren jika dan hanya jika BD CE AF = 1. DC EA F B Bukti. Teorema di atas membutuhkan pembuktian ”dua arah”, yaitu: jika AD, BE, CF konkuren, maka kesamaan di atas berlaku dan jika kesamaan berlaku, maka AD, BE, CF konkuren. A

A

F'

E

F P

B

D

E

F

C

B

P'

D

C

Gambar 1. Pertama, kita buktikan dulu bahwa jika AD, BE, CF konkuren maka kesamaan yang diberikan berlaku. Misalkan P adalah titik perpotongan ketiga garis AD, BE, CF . Perhatikan dua identitas berikut: [ABD] BD [P BD] BD = dan = , DC [ACD] DC [P CD] yang diperoleh dari fakta bahwa jika dua buah segitiga memiliki ”tinggi” yang sama, maka perbandingan luasnya sama dengan perbandingan ”alas”-nya. Dari dua identitas tersebut, kemudian kita peroleh [ABD] − [P BD] [AP B] BD = = (Mengapa?). DC [ACD] − [P CD] [CP A] 37

Dengan cara yang sama, kita peroleh [BP C] AF [CP A] CE = dan = . EA [AP B] FB [BP C] Jadi, [AP B] [BP C] [CP A] BD CE AF = = 1. DC EA F B [CP A] [AP B] [BP C] Sekarang misalkan kesamaan di atas berlaku. Akan dibuktikan bahwa AD, BE, CF berpotongan di satu titik. Untuk membuktikan hal ini, kita menggunakan teknik titik bayangan (phantom point). Perhatikan Gambar 1 sebelah kanan. Misalkan cevian AD dan BE berpotongan di titik P  dan garis CP  memotong sisi AB di titik F  . Kita cukup membuktikan bahwa F  = F , atau dengan kata lain, kedua titik tersebut berimpit (Mengapa?). Untuk membuktikan hal ini, pertama perhatikan bahwa tiga cevian AD, BE, CF  konkuren (bertemu di titik F  ). Dengan demikian, kita punya BD CE AF BD CE AF  =1= ,  DC EA F B DC EA F B sehingga

AF  AF = .  FB FB Dari sini kita simpulkan F = F  (Mengapa?) dan kita selesai.

Pada kasus-kasus tertentu, teorema Ceva di atas lebih mudah digunakan dalam bentuk trigonometri berikut: Akibat (”Trig Ceva”). Misalkan ABC sebuah segitiga dan P, Q, R tiga titik yang berturut-turut terletak pada sisi-sisi BC, CA, AB. Maka garis-garis AP, BQ, CR konkuren jika dan hanya jika sin ∠CAP sin ∠ABQ sin ∠BCR = 1. sin ∠AP B sin ∠QBC sin ∠RCA Akibat di atas dapat dibuktikan dengan mudah dengan menggunakan aturan sinus dan teorema Ceva atau secara langsung dengan menggunakan aturan sinus pada beberapa segitiga. Bukti selengkapnya diserahkan kepada pembaca Tiga buah titik dikatakan kolinear jika terletak pada satu garis. Kriteria kolinearitas tiga titik yang berada pada ketiga sisi-sisi segitiga diberikan oleh Menelaos. 38

Teorema Menelaos. Misalkan ABC sebuah segitiga dan D, E, F tiga titik pada garis-garis BC, CA, AB (D, E, F bisa terletak pada perpanjangan sisi-sisi segitiga ABC). Maka D, E, F kolinear jika dan hanya jika BD CE AF = 1. DC EA F B Bukti. Pertama, kita buktikan bahwa jika D, E, F kolinear, maka kesamaan yang diberikan berlaku. Buat garis tegak lurus dari tiga A, B, C terhadap garis yang melalui D, E, F dan misalkan P , Q, R adalah ketiga kaki tegaklurusnya (perhatikan Gambar 2). A Q F P

E R

B

C

D

Gambar 2. Kita punya tiga kesamaan berikut yang dapat diperoleh dengan meninjau kesebangunan-kesebangunan beberapa segitiga: BQ CE CR AF AP BD = , = , dan = . DC CR EA AP FB BQ Dengan mengalikan ketiga kesamaan tersebut, kita peroleh BQ CR AP BD CE AF = = 1. DC EA F B CR AP BQ Bukti untuk arah yang satunya (yaitu jika kesamaan berlaku, maka ketiga titik kolinear) dapat dibuktikan dengan menggunakan titik bayangan, sama seperti bukti teorema Ceva. Hal ini dilakukan dengan memisalkan F  sebagai perpotongan garis-garis AB dan DE lalu membuktikan bahwa F = F  . Bukti selengkapnya diserahkan kepada pembaca.

39

1.2

Panjang Cevian: Teorema Stewart

Panjang cevian dapat dihitung dengan menggunakan teorema berikut: Teorema Stewart. Misalkan AX adalah sebuah cevian dengan panjang p yang membagi sisi BC menjadi dua segmen, yaitu BX dengan panjang m dan XC dengan panjang n. Maka berlaku a(p2 + mn) = b2 m + c2 n. Bukti. A

p

c

B

m

X

b

n

C

Gambar 3. Dengan menggunakan aturan cosinus pada segitiga ABX dan ACX, kita peroleh p 2 + n 2 − b2 p2 + m2 − c2 dan cos ∠AXC = . cos ∠AXB = 2pm 2pn Karena ∠AXB = 180◦ − ∠AXC, maka cos ∠AXB = − cos ∠AXC atau setara dengan cos ∠AXB + cos ∠AXC = 0. Dengan demikian, kita punya p2 + m 2 − c 2 p 2 + n 2 − b2 + = 0, 2pm 2pn yang setara dengan n(p2 + m2 − c2 ) + m(p2 + n2 − b2 ) = 0, atau setara juga dengan (m + n)(p2 + mn) = b2 m + c2 n dan setara dengan kesamaan yang diinginkan, karena m + n = a. Dengan teorema di atas, panjang cevian AX dapat dihitung secara langsung, yaitu  b2 m + c2 n − mn. p= a 40

1.3

Titik-titik dan Garis-garis Istimewa pada Segitiga

Pada sebuah segitiga, terdapat banyak titik dan garis istimewa, namun dalam subbab ini, hanya akan dijelaskan beberapa diantaranya, yaitu garis berat, titik berat, garis tinggi, titik tinggi, garis bagi sudut, titik pusat lingkaran dalam, garis sumbu dan titik pusat lingkaran luar. 1.3.1

Garis Berat (Median) dan Titik Berat (Centroid)

Kita mulai dengan definisi garis berat. Garis berat sebuah segitiga adalah garis yang melalui titik sudut segitiga tersebut dan titik tengah sisi di depannya. Dengan demikian, setiap segitiga akan memiliki tiga garis berat. Dengan menggunakan teorema Ceva, teorema berikut dapat dibuktikan dengan mudah: Teorema. Misalkan ABC sebuah segitiga dan A , B  , C  titik-titik tengah sisisisi BC, CA, AB. Maka garis-garis berat AA , BB  , CC  konkuren. A

C'

B' G

B

A'

C

Gambar 4. Titik potong ketiga garis berat sebuah segitiga kemudian dinamakan titik berat. Karena berpotongan di satu titik, maka ketiga garis berat sebuah segitiga akan membagi segitiga tersebut menjadi enam bagian. Selain itu, setiap garis berat akan membagi garis berat lainnya menjadi dua bagian. Kita punya teorema berikut: Teorema. Misalkan AA , BB  , CC  adalah tiga garis berat segitiga ABC yang berpotongan di titik berat G. Maka keenam segitiga AGB  , AGC  , BGA , BGC  , CGA dan CGB  memiliki luas yang sama. Bukti. Perhatikan kembali Gambar 4. Karena A , B  , C  berturut-turut adalah titik-titik tengah BC, CA, AB, kita punya bahwa [BGA ] = [CGA ],

41

[AGB  ] = [CGB  ], dan [AGC  ] = [BGC  ]. Kita juga punya bahwa [ABA ] = [ACA ], sehingga 2[AGC  ] = [ABG] = [ABA ] − [BGA ] = [ACA ] − [CGA ] = [ACG] = 2[AGB  ], sehingga [AGC  ] = [AGB  ]. Dengan cara yang sama diperoleh [BGA ] = [AGB  ] = [AGC  ], dan kesimpulan mengikuti. Teorema. Misalkan AA , BB  , CC  adalah tiga garis berat segitiga ABC yang berpotongan di titik berat G. Maka AG BG CG = = = 2.   GA GB GC  Bukti. Kita gunakan lagi teorema sebelumnya. Kita punya bahwa AG [ABG] [AGC  ] + [BGC  ] = = = 2, GA [GBA ] [GBA ] karena [AGC  ] = [BGC  ] = [GBA ]. Kesamaan lain dapat diperoleh dengan cara yang sama. Panjang garis berat sendiri dapat dihitung dengan mudah menggunakan teorema Stewart. 1.3.2

Garis Bagi Sudut (Bisector)

Garis yang membagi sebuah sudut segitiga menjadi dua bagian yang sama besar dinamakan garis bagi sudut. Pertama, kita punya teorema berikut: Teorema. Misalkan AA , BB  , CC  adalah garis-garis bagi sudut segitiga ABC (dengan demikian, ∠A AB = ∠A AC = 12 A, dan seterusnya). Maka A B c BC a C A b = , = , dan = .    AC b BA c CB a Bukti.

42

A

B

A'

C

Gambar 5. Dengan aturan sinus pada segitiga-segitiga AA B dan AA C, kita peroleh sin ∠A AB A C sin ∠A AC A B = dan = . AB sin ∠AA B AC sin ∠AA C Karena ∠A AB = ∠A AC dan sin ∠AA B = 180◦ −sin ∠AA C, maka sin ∠A AB = sin ∠A AC dan sin ∠AA B = sin ∠AA C. Akibatnya, A B sin ∠A AB sin ∠A AC A C = = = , AB sin ∠AA B sin ∠AA C AC sehingga kita peroleh

A B AB c = = .  AC AC b Dua kesamaan sisanya dapat dibuktikan dengan cara yang sama. Dengan menggunakan teorema di atas dan teorema Ceva atau langsung dengan menggunakan Trig Ceva, kita peroleh teorema berikut: Teorema. Misalkan AA , BB  , CC  adalah tiga garis bagi sudut segitiga ABC. Maka ketiga garis tersebut konkuren.

Selain itu, karena perbandingan A B/A C dan A B + A C dapat dinyatakan dalam panjang sisi-sisi a, b, c, maka panjang A B dan A C juga dapat dinyatakan dalam a, b, c. Selanjutnya, dengan teorema Stewart, panjang garis bagi sudut AA juga dapat dihitung. Perhitungan ini diserahkan kepada pembaca sebagai latihan. 1.3.3

Garis Tinggi (Altitude) dan Titik Tinggi (Orthocenter)

Garis tinggi sebuah segitiga adalah garis yang melalui titik sudut sebuah segitiga dan tegak lurus dengan sisi di depannya. Kita punya teorema berikut 43

Teorema. Misalkan AA , BB  , CC  garis-garis tinggi sebuah segitiga (dengan demikian, AA tegak lurus BC, dan seterusnya). Maka ketiga garis tersebut konkuren. Bukti. A C' B'

B

A'

C

Gambar 6. Kasus dimana ABC segitiga siku-siku trivial (sangat jelas dan tidak ada yang perlu dibuktikan), karena ketiga garis tinggi akan berpotongan pada titik sudut siku-siku segitiga ABC). Jadi kita cukup meninjau dimana ABC bukan segitiga siku-siku. Sekarang akan kita buktikan teorema tersebut untuk kasus dimana ABC segitiga lancip. Kita punya bahwa AA AA = tan B dan = tan C. BA CA Dengan demikian,

BA tan C . =  CA tan B Dengan cara yang sama, diperoleh CB  AC  tan A tan B dan . = =   AB tan C BC tan A Akibatnya,

BA CB  AC  tan C tan A tan B = = 1,    AC BAC B tan B tan C tan A sehingga AA , BB  , CC  konkuren. Kasus terakhir dimana segitiga ABC tumpul diserahkan kepada pembaca. Satu hal yang perlu diperhatikan dalam pembuktian kasus tersebut adalah bahwa titik perpotongan ketiga garis tinggi terletak di luar segitiga ABC. 44

Titik potong ketiga ketiga garis tinggi sebuah segitiga selanjutnya disebut sebagai titik tinggi (orthocenter) segitiga. Kemudian, sama seperti pada garis bagi sudut, panjang BA dan CA dapat dinyatakan dalam panjang sisi-sisi a, b, c dan fungsi trigonometri sudut-sudut A, B, C. Dengan menyatakan fungsi trigonometri sudut dalam panjang sisi (misalnya dengan aturan cosinus), panjang BA dan CA dapat dinyatakan dalam panjang sisi-sisi a, b, c. Selanjutnya, teorema Stewart dapat digunakan untuk menghitung panjang garis tinggi AA . 1.3.4

Garis Sumbu (Perpendicular Bisector)

Garis sumbu sebuah segitiga adalah garis yang melalui titik tengah sebuah sisi dan tegak lurus terhadap sisi tersebut. Kita punya teorema berikut: Teorema. Misalkan lA , lB , lC adalah garis-garis sumbu segitiga ABC yang berturut-turut tegak lurus terhadap sisi-sisi BC, CA, AB (dengan demikian, lA melalui titik tengan BC, dan seterusnya). Maka lA , lB , lC konkuren. Bukti. Kita tidak dapat menggunakan teorema Ceva untuk membuktikan teorema ini karena garis-garis sumbu sebuah segitiga bukan merupakan cevian. Untuk membuktikan teorema ini, kita cukup membuktikan bahwa titik potong dua buah garis terletak pada garis yang ketiga (Mengapa?). A

B' lB O B

lA A'

C

Gambar 7. Misalkan A , B  , C  berturut-turut adalah titik-titik tengah sisi-sisi BC, CA, AB. Misalkan juga O adalah perpotongan garis lA dan lB . Sekarang akan dibuktikan bahwa O terletak pada garis lC . Pertama, tinjau segitiga OA B dan OA C. Dengan teorema Pythagoras pada kedua segitiga tersebut dan karena A adalah titik tengah sisi BC, kita punya √ √ OB = A B 2 + A O2 = A C 2 + A O2 = OC. Dengan cara yang sama, kita punya bahwa OC = OA. Jadi, kita punya OA = OB, sehingga OAB segitiga sama kaki. Oleh karena itu, garis tinggi segitiga OAB dari 45

titik O akan memotong titik tengah AB, atau dengan kata lain O terletak pada garis yang melalui titik tengah AB dan tegak lurus garis AB, yaitu garis lC . Hal ini melengkapkan pembuktikan.

1.4

Lingkaran Dalam (incircle) dan Lingkaran Luar Segitiga (circumcircle)

Lingkaran dalam segitiga adalah lingkaran yang menyinggung ketiga sisi segitiga dari dalam dan lingkaran luar segitiga adalah lingkaran yang melalui ketiga titiktitik sudut segitiga. Kita punya beberapa teorema berikut mengenai titik pusat lingkaran dalam dan luar serta panjang jari-jarinya. Teorema. Titik perpotongan ketiga garis bagi sudut segitiga ABC adalah titik pusat lingkaran dalam segitiga ABC dan panjang jari-jarinya sama dengan [ABC]/s. Bukti. A

C'

R Q I

B

A'

P

B'

C

Gambar 8. Misalkan AA , BB  dan CC  adalah ketiga garis bagi sudut segitiga ABC yang berpotongan di titik I. Misalkan juga P , Q, R berturut-turut adalah kaki tegak lurus titik I pada sisi-sisi BC, CA, AB, atau dengan kata lain, P , Q, R terletak pada sisi-sisi BC, CA, AB sedemikian hingga IP tegak lurus BC, IQ tegak lurus CA dan IR tegak lurus AB. Karena ∠QAI = ∠CAA = ∠BAA = ∠RAI dan ∠AQI = 90◦ = ∠ARI maka kedua segitiga siku-siku AQI dan ARI sebangun. Kemudian karena sisi miring kedua segitiga siku-siku tersebut berimpit (sehingga sama panjang), maka kedua segitiga tersebut sebangun. Oleh karenanya, kita punya bahwa IQ = IR. Dengan cara yang sama, diperoleh juga bahwa IP = IQ. Jadi kita peroleh IP = IQ = IR. Akibatnya, lingkaran dengan pusat I dan berjari-jari IP = IQ = IR menyinggung sisi-sisi BC, CA, AB. Lingkaran tersebut 46

kemudian disebut sebagai lingkaran dalam segitiga (incenter) ABC dan jari-jarinya (IP = IQ = IR) disebut sebagai jari-jari lingkaran dalam segitiga (inradius) ABC. Sekarang misalkan r menyatakan panjang jari-jari lingkaran dalam segitiga ABC. Karena segitiga AQI dan ARI kongruen maka AQ = AR dan 1 1 [AQIR] = 2 × [AQI] = 2 × · AQ · IQ = (AQ + AR) · r. 2 2 Dengan cara yang sama, diperoleh 1 1 [BRIP ] = (BR + BP ) · r dan [CP IQ] = (CP + CQ) · r. 2 2 Akibatnya, [ABC] = [AQIR] + [BRIP ] + [CP IQ] 1 1 1 = (AQ + AR) · r + (BR + BP ) · r + (CP + CQ) · r 2 2 2 1 (AR + BR + BP + CP + CQ + AQ) · r = 2 1 (AB + BC + CA) · r = 2 = sr, atau setara dengan r = [ABC]/s. Teorema. Titik perpotongan ketiga garis sumbu segitiga ABC adalah titik pusat lingkaran luar segitiga ABC dan panjang jari-jarinya sama dengan abc/4[ABC]. Bukti. A

C'

B' lB O

B

lA A'

47

C

Gambar 9. Misalkan lA , lB , lC adalah ketiga garis sumbu segitiga ABC dan O adalah perpotongan ketiga garis tersebut. Dengan cara yang sama seperti bukti teorema konkurensi garis sumbu, kita peroleh bahwa OA = OB = OC. Akibatnya, O adalah titik pusat lingkaran yang melalui ketiga titik sudut segitiga ABC (yaitu lingkaran yang berpusat di O dengan panjang jari-jari OA = OB = OC). Lingkaran tersebut selanjutnya dinamakan lingkaran luar segitiga (circumcircle) ABC dan titik pusatnya disebut titik pusat lingkaran luar segitiga (circumcenter) ABC. Sekarang misalkan AL adalah sebuah garis tinggi segitiga ABC, sehingga 1 AL = sin B dan [ABC] = × BC × AL. AB 2 Jadi, 1 1 1 × BC × AL = × BC × AB sin B = ac sin B. 2 2 2 Selanjutnya dengan aturan sinus, kita punya bahwa [ABC] =

b = 2R, sin B sehingga 1 b abc 1 = , [ABC] = ac sin B = ac × 2 2 2R 4R atau setara dengan R=

2 2.1

abc . 4[ABC]

Lingkaran dan Beberapa Sifatnya Kuasa Titik Terhadap Lingkaran (Power of a Point with Respect to a Circle)

Pembahasan pertama di subbab ini adalah tentang hubungan sebuah titik dengan sebuah lingkaran. Di sini kita mengenal konsep kuasa sebuah titik terhadap sebuah lingkaran. Teorema. Diberikan sebuah lingkaran dan sebuah titik P . Buat sebuah garis yang memotong lingkaran di dua titik A dan B (jika garis tersebut menyinggung lingkaran, maka A = B). Maka hasil kali P A×P B bernilai tetap, tidak tergantung oleh garis yang dibuat. 48

Bukti. Buat garis lain yang melalui titik P dan memotong lingkaran di dua titik C dan D. Sekarang perhatikan bahwa ∠P BC = ∠P DA dan ∠BP C = ∠DP A. B A

A

D P

P

C

C

B D

Gambar 10. Dengan demikian, kedua segitiga P BC dan P DA sebangun, sehingga PC PB = , PD PA yang setara dengan P A × P B = P C × P D. Sekarang kita gunakan notasi yang sama dengan notasi pada teorema di atas. Misalkan O dan r berturut-turut adalah tiik pusat dan panjang jari-jari lingkaran tersebut. Tinjau garis yang melalui O dan P yang memotong lingkaran di dua titik R dan S. Nilai (OP − r)(OP + r) = OP 2 − r2 disebut sebagai kuasa titik P terhadap lingkaran tersebut. Jika P berada di dalam lingkaran, maka kuasanya negatif, jika P di luar maka kuasanya positif dan jika P terletak pada lingkaran, kuasanya nol. A

B

R

A P

P

R O

O S B S

Gambar 11. 49

Terkait dengan teorema sebelumnya, jika P berada di luar atau pada lingkaran, maka P A × P B = P R × P S = (OP − r)(OP + r) = OP 2 − r2 sama dengan kuasa P terhadap lingkaran tersebut dan jika P di dalam lingkaran, maka P A × P B = P R × P S = (r − OP )(r + OP ) = r2 − OP 2 sama dengan −1 kali kuasa P terhadap lingkaran tersebut.

2.2

Jarak Titik Pusat Lingkaran Dalam dan Luar Segitiga: Teorema Euler

Teorema kuasa titik terhadap lingkaran di atas dapat digunakan untuk membuktikan salah satu teorema penting dalam geometri: Teorema Euler. Teorema Euler. Misalkan I dan O berturut-turut adalah titik-titik pusat lingkaran dalam dan luar segitiga ABC. Jika r dan R berturut-turut menyatakan panjang jari-jari lingkaran dalam dan luar segitiga ABC, maka OI 2 = R2 − 2rR. Bukti. M

A D D

N

I

O

E B

E D

C

L

Gambar 12. Misalkan N adalah kaki tegak lurus I pada sisi CA, sehingga IN = r dan garis bagi sudut AI memotong lingkaran luar segitiga ABC di titik L. Karena ∠BAL = ∠CAL, maka L adalah titik tengah busur BC yang tidak memuat A. 50

Sekarang misalkan garis LO memotong lingkaran luar segitiga ABC lagi di titik M , sehingga LM adalah diameter lingkaran tersebut yang tegak lurus dengan BC (Mengapa?). Sekarang misalkan α = 12 A dan β = 12 B. Kita punya bahwa ∠BM L = ∠BAL = α,sehingga LB IN r LB = = sin α = = , 2R LM IA IA sehingga LB = 2R sin α dan IA = r/ sin α. Kita juga punya bahwa ∠LBC = ∠LAC = α, sehingga ∠LBI = α + β = 180◦ − ∠AIB = ∠LIB, sehingga LBI adalah segitiga sama kaki, sehingga LB = LI. Sekarang karena I berada di dalam lingkaran luar segitiga ABC, maka LI × IA sama dengan −1 kali kuasa I terhadap lingkaran luar segitiga ABC, yaitu R2 − OI 2 . Di sisi lain, kita punya bahwa LI × IA = LB × IA = 2R sin α ×

r = 2rR, sin α

sehingga R2 − OI 2 = 2rR yang setara dengan kesamaan yang ingin dibuktikan. Sebagai akibat dari teorema tersebut, kita punya ketaksamaan berikut Akibat. Jika R dan r berturut-turut menyatakan panjang jari-jari lingkaran luar dan dalam suatu segitiga, maka R ≥ 2r.

2.3

Segiempat Talibusur dan Beberapa Sifatnya

Segiempat talibusur adalah segiempat yang keempat titik sudutnya terletak pada satu lingkaran. Berikutnya akan dijelaskan beberapa kriteria dan sifat segiempat talibusur. 2.3.1

Beberapa Kriteria Segiempat Talibusur

Misalkan ABCD sebuah segiempat talibusurdan O adalah  titik pusat lingkaran 1 luarnya. Kita punya bahwa ∠ADB = ∠ACB = 2 ∠AOB dan ∠ADC+∠ABC = 180◦ . Konvers dari pernyataan tersebut ternyata berlaku. Kita punya teorema berikut: 51

Teorema. Jika ABCD sebuah segiempat konveks (yaitu kedua diagonalnya terletak di dalam segiempat) dan ∠ADB = ∠ACB, maka ABCD segiempat talibusur. Bukti. Kita menggunakan teknik titik bayangan lagi. Misalkan lingkaran luar segitiga ABC memotong garis BD di titik D (dalam hal ini, D adalah titik bayangan dari titik D). Selanjutnya, kita cukup membuktikan bahwa D = D (Mengapa?). B

B

A

C

A

C D

D'

D'

D

Gambar 13. Perhatikan bahwa ABCD adalah segiempat talibusur, sehingga kita punya ∠ACB = ∠AD B.Dan karena ∠ACB = ∠ADB, maka ∠ADB = ∠AD B. Akibatnya, D = D (Mengapa?) dan kita selesai. Teorema. Jika ABCD sebuah segiempat dan ∠ADC + ∠ABC = 180◦ , maka ABCD segiempat talibusur. Teorema tersebut dapat dibuktikan dengan menggunakan konsep titik bayangan dan pembuktian ini diserahkan kepada pembaca. Selain dua kriteria dasar di atas, konsep titik bayangan dan teorema kuasa titik terhadap lingkaran dapat digunakan untuk membuktian kriteria segiempat talibusur berikut. Teorema. Misalkan ABCD sebuah segiempat talibusur. Misalkan juga AD dan BC berpotongan di E dan AB dan CD berpotongan di F . Jika salah satu dari dua kesamaan berikut: EA × EC = EB × ED atau F A × F B = F C × F D, maka ABCD adalah segiempat talibusur. 52

2.3.2

Teorema Ptolemeus dan Brahmagupta

Kita akhiri subbab ini dengan dua teorema tentang segiempat talibusur. Teorema pertama menyebutkan hubungan antara panjang diagonal dan panjang sisi-sisi segiempat talibusur dan teorema berikutnya tentang hubungan luas segiempat talibusur dengan panjang sisi-sisi nya. Teorema. Misalkan ABCD sebuah segiempat talibusur. Maka AB × CD + BC × AD = AC × BD. Bukti. C

B

D E

A

Gambar 14. Misalkan E titik pada segmen AC sehingga ∠ABE = ∠DBC. Karena ∠BAE = ∠BAC = ∠BDC, maka segitiga ABE sebangun dengan segitiga DBC, sehingga AE CD = AB BD atau setara dengan AE × BD = AB × CD. Sekarang perhatikan bahwa ∠CBE = ∠ABC − ∠ABE = ∠ABC − ∠DBC = ∠DBA dan ∠BCE = ∠BCA = ∠BDA. Dengan demikian, segitiga BCE sebangun dengan segitiga BDA, sehingga AD CE = BC BD atau setara dengan CE · BD = BC · AD. Jadi, AC × BD = AE × BD + CE × BD = AB × CD + BC × AD. 53

Teorema berikut ini diperoleh oleh Brahmagupta, seorang matematikawan India pada abad ketujuh A.D. Teorema. Misalkan ABCD sebuah segiempat talibusur dengan panjang sisisisi AB = a, BC = b, CD = c, DA = d dan s = (a + b + c + d)/2. Maka luas segiempat talibusur tersebut adalah  [ABCD] = (s − a)(s − b)(s − c)(s − d) Bukti. A B D

D

SD

C

Gambar 15. Salah satu cara termudah membuktikan teorema di atas adalah menggunakan trigonometri. Misalkan ∠ABC = α, sehingga ∠ADC = 180◦ −α. Sekarang dengan aturan cosinus pada segitiga ABC dan ADC, kita peroleh AC 2 = a2 + b2 − 2ab cos α dan AC 2 = c2 + d2 − 2cd cos(180◦ − α). Karena cos(180◦ − α) = − cos α, dari dua kesamaan di atas kita peroleh a2 + b2 − 2ab cos α = c2 + d2 + 2cd cos α, yang setara dengan 2(ab + cd) cos α = a2 + b2 − c2 − d2 , sehingga

 2

cos α =

a2 + b2 − c2 − d2 2(ab + cd)

2

Sekarang perhatikan bahwa 1 1 1 [ABC] = ab sin α dan [ACD] = cd sin(180◦ − α) = cd sin α, 2 2 2 54

sehingga 1 [ABCD] = [ABC] + [ACD] = (ab + cd) sin α. 2 Dengan demikian 1 1 (ab + cd)2 sin2 α = (ab + cd)2 (1 − cos2 α) 4 4    2 2 2 2 2 1 a +b −c −d (ab + cd)2 1 − = 4 2(ab + cd)

[ABCD]2 =

=

1 1 (ab + cd)2 − (a2 + b2 − c2 − d2 )2 . 4 16

Di sisi lain, dengan menjabarkan kedua ruas, kita juga punya bahwa 1 1 (s − a)(s − b)(s − c)(s − d) = (ab + cd)2 − (a2 + b2 − c2 − d2 )2 . 4 16 Kesimpulan selanjutnya mengikuti. Sebuah kasus khusus dimana titik D berimpit dengan titik A atau C akan menghasilkan teorema Heron. Teorema. Misalkan ABC sebuah segitiga dengan panjang sisi-sisi a, b, c dan s = (a + b + c)/2. Maka  [ABC] = s(s − a)(s − b)(s − c).

3

Segiempat

Tidak banyak hal yang bisa dibahas dalam segiempat secara umum. Pada beberapa kasus, sebuah segiempat lebih mudah dipandang sebagai dua buah segitiga yang memiliki satu sisi persekutuan.

3.1

Jajaran Genjang Varignon

Pembahasan pertama ini adalah mengenai bangun datar yang dibentuk oleh titiktitik tengah keempat sisi sebuah segiempat. Kita memiliki teorema berikut: Teorema. Jika K, L, M , N adalah titik-titik tengah sisi-sisi AB, BC, CD, DA pada segiempat ABCD, maka KLM N adalah jajaran genjang. Jajaran genjang yang terbentuk tersebut disebut sebagai jajaran genjang Varignon. Lebih jauh, luas jajaran genjang Varignon sama dengan setengah luas segiempatnya. 55

Bukti. B

K L A

N C M

D

Gambar 16. Perhatikan segitiga ABC. Karena K dan L berturut-turut adalah titik-titik tengah AB dan BC, maka KL tegak lurus AC. Dengan cara yang sama, kita punya bahwa M N juga tegak lurus dengan AC. Akibatnya, KL sejajar dengan AC. Dengan cara yang sama juga, kita peroleh KN sejajar LM . Kesimpulan mengikuti dengan mudah. Untuk luasnya, kita punya bahwa [KLM N ] = [ABCD] − [AKN ] − [BKL] − [CLM ] − [DM N ] 1 1 1 1 = [ABCD] − [ABD] − [ABC] − [BCD] − [CDA] 4 4 4 4 1 1 = [ABCD] − [ABCD] − [ABCD] 4 4 1 [ABCD]. = 2

3.2

Lingkaran Dalam Segiempat

Tidak semua segiempat memiliki lingkaran dalam, yaitu suatu lingkaran di dalam segiempat yang menyinggung keempat sisi segiempat. Teorema berikut memberikan sifat sebuah segiempat yang memiliki lingkaran dalam Teorema. Misalkan ABCD adalah segiempat konveks yang memiliki lingkaran dalam. Maka AB + CD = BC + DA. Bukti. Misalkan lingkaran dalam segiempat ABCD menyinggung sisi-sisi AB, BC, CD, DA berturut-turut di titik-titik K, L, M , N .

56

B

K

A

N

L

D

M

C

Gambar 17. Dengan demikian, kita punya AK = AN , BK = BL, CL = CM , dan DM = DN sehingga AB + CD = AK + BK + CM + DM = AN + BL + CL + DN = BC + DA. Konvers dari teorama di atas ternyata berlaku. Kita punya teorema berikut. Teorema. Misalkan ABCD adalah segiempat konveks yang memenuhi AB + CD = BC + DA. Maka ABCD memiliki lingkaran dalam. Bukti. B K

A

P

L D

D'

M C

Gambar 18. 57

Misalkan garis AB dan CD berpotongan di titik P dan Γ adalah lingkaran dalam segitiga P BC. Misalkan juga garis melalui A yang menyinggung Γ memotong P C di titik D sehingga Γ juga merupakan lingkaran dalam segiempat ABCD . Selanjutnya, cukup dibuktikan bahwa D = D . Dengan menggunakan teorema sebelumnya, kita punya AB +CD = BC +D A dan menggunakan kesamaan yang diberikan, kita punya CD − CD = AB + CD − (AB + CD) = BC + D A − (BC + DA) = D A − DA. Jika CD > CD, maka DD = CD − CD = D A − DA, sehingga D A = DA + DD yang jelas tidak mungkin karena menurut ketaksamaan segitiga D A < DA + DD . Demikian juga halnya dengan CD < CD. Kita simpulkan bahwa CD = CD, sehingga D = D dan kita selesai.

4 4.1

Lampiran Rumus-Rumus Trigonometri

Berikut diberikan rangkuman rumus-rumus trigonometri terutama untuk sinus dan cosinus. Rumus untuk tangent dan cotangent dapat diturunkan dari rumus-rumus tersebut. Pertama adalah rumus trigonometri untuk jumlah maupun selisih dua sudut dan rumus untuk mengubah jumlah atau selisih sinus dan cosinus menjadi hasil kali sinus dan cosinus. cos(A + B) cos(A − B) sin(A + B) sin(A − B)

= = = =

sin A + sin B = sin A − sin B = cos A + cos B = cos A − cos B =

cos A cos B − sin A sin B cos A cos B + sin A sin B sin A cos B + cos A sin B sin A cos B − cos A sin B     A−B A+B cos 2 sin 2 2     A−B A+B sin 2 cos 2 2     A−B A+B cos 2 sin 2 2     A−B A+B sin . −2 sin 2 2 58

Selanjutnya adalah rumus trigonometri untuk dua kali sudut atau setengah kali sudut.

sin 2A = 2 sin A cos A cos 2A = 2 cos2 A − 1 = 1 − 2 sin2 A 2 tan A tan 2A = 1 − tan2 A A 1 − cos A = ± sin 2 2  1 + cos A A cos = ± . 2 2

4.2

Perluasan Aturan Sinus dan Aturan Cosinus

Teorema (Aturan Sinus). Misalkan ABC segitiga dengan panjang jari-jari lingkaran luar R. Maka a b c = = = 2R. sin A sin B sin C Teorema (Aturan Cosinus). Misalkan ABC segitiga. Maka a2 = b2 + c2 − 2bc cos A b2 = c2 + a2 − 2ca cos B c2 = a2 + b2 − 2ab cos C.

59

Teorema Misalkan x1, x2, … , xn adalah n obyek yang berbeda. Maka banyaknya n  permutasi adalah n! permutasi. Contoh 4 Banyaknya permutasi dari huruf-huruf dari kata RAMBUT adalah 6! = 720. Contoh 5 Sebuah lemari memuat 5 buku berbahasa Jerman, 7 buku berbahasa Spanyol, dan 8 buku berbahasa Indonesia. Diketahui bahwa tidak ada dua buku yang sama. a. Berapa banyak penyusunan berbeda yang bisa dilakukan pada buku-buku ini? b. Berapa banyak penyusunan berbeda yang bisa dilakukan pada buku-buku ini jika buku-buku dari masing-masing bahasa harus diletakkan saling berdekatan? c. Berapa banyak penyusunan berbeda yang bisa dilakukan pada buku-buku ini jika semua buku berbahasa Indonesia harus saling berdekatan? d. Berapa banyak penyusunan berbeda yang bisa dilakukan pada buku-buku ini jika tidak boleh ada dua buku berbahasa Indonesia diletakkan saling berdekatan? Jawab: a. Perhatikan bahwa kita melakukan permutasi pada 5+7+8 = 20 buku. Maka banyaknya kemungkinan penyusunan adalah 20!. b. Buku-buku dengan bahasa yang sama di’ikat’ sehingga saling berdekatan. Perhatikan bahwa kita melakukan permutasi pada 3 bahasa, yakni dengan 3! cara. Kemudian, kita lakukan permutasi pada buku-buku berbasa Jerman dalam 5! cara, pada buku-buku berbahasa Spanyol dalam 7! cara, dan pada buku-buku berbahasa Indonesia dengan 8! cara. Jadi total banyaknya cara penyusunan buku adalah 3!5!7!8!. c. Pertama-tama, kita atur dulu buku-buku berbahasa Jerman dan Spanyol. Perhatikan dengan mengatur 5 + 7 = 12 buku-buku ini, kita mendapatkan 13 ’ruang’, yakni ruang sebelum buku pertama, ruang di antara buku-buku, dan ruang setelah buku terakhir. Untuk memastikan bahwa buku-buku berbahasa Indonesia saling berdekatan, bukubuku ini kita ’ikat’ bersama-sama dan diletakkan pada salah satu ’ruang’. Perhatikan bahwa buku-buku berbahasa Indonesia dapat disusun dengan 8! cara, sedangkan buku-buku berbahasa Jerman dan Perancis dengan 12! cara. Sehingga total banyaknya cara adalah 13˜8!12!. d. Pertama-tama atur buku berbahasa Jerman dan Spanyol. Dengan mengatur 5 + 7 = 12 buku ini, diperoleh 13 ’ruang’. Untuk memastikan bahwa tidak ada buku-buku berbahasa Indonesia yang saling berdekatan, kita letakkan buku-buku tersebut ke dalam ’ruang-ruang’ ini. Buku pertama dapat ditempatkan ke salah satu dari 13 ruang. Buku kedua dapat ditempatkan ke sisa 12 ruang yang ada, buku ketiga dapat ditempatkan ke sisa 11 ruang yang ada, dan seterusnya, hingga buku ke 8 dapat ditempatkan ke sisa 6 ruang yang ada. Jadi total banyaknya pengaturan yang ada adalah 13 · 12 · 11 · 10 · 9 · 8 · 7 · 6 · 12!. Latihan 6 Sebuah rak buku terdiri atas 3 novel Rusia, 4 novel Jerman, dan 5 novel Indonesia. Ada berapa banyak cara penyusunan novel-novel ini jika, a. tidak ada batasan pengelompokkan novel b. semua novel Indonesia harus berdekatan c. tidak ada novel Indonesia yang saling berdekatan Latihan 7 Berapa banyak kemungkinan permutasi untuk kata LEMARI? Berapakah banyak permutasi yang diawali dengan huruf R dan diakhiri dengan huruf E? Berapa banyak

61

permutasi jika P dan U selalu bersama-sama dengan urutan PU? Berapa banyak permutasi yang ada jika tidak ada huruf vokal (A, E, I) yang berdekatan? Latihan 8 Berapa banyak pengaturan yang bisa dilakukan pada huruf-huruf dari kata TERANG jika huruf N dan G tidak pernah terpisah? Latihan 9 (AIME 1991) Diketahui sebuah bilangan rasional, tuliskan sebagai perbandingan paling sederhana, kemudian kalikan penyebut dengan pembilang. Tentukan berhapa banyak bilangan rasional yang nilainya di antara 0 dan 1 dimana 20! menjadi hasil kalinya? Latihan 10 Seekor laba-laba mempunyai sebuah kaos kaki dan sebuah sepatu untuk masingmasing dari delapan kakinya. Tentukan banyaknya kemungkinan urutan laba-laba bisa menggunakan kaos kaki dan sepatu, dengan asumsi bahwa, untuk setiap kaki, kaos kaki harus digunakan sebelum sepatu? Latihan 11 Ada berapa banyak carakah 8 orang bisa ditempatkan dalam sebuah kursi panjang, jika: a. tidak ada batasan dalam pengaturan posisi duduk b. orang X dan Y harus duduk berdampingan c. ada 4 pria dan 4 wanita dimana tidak ada 2 pria atau 2 wanita yang duduk berdampingan d. ada 4 pasang suami-istri di mana setiap pasang suami-istri harus duduk berdampingan e. ada 4 pria dan mereka harus duduk berdekatan.

1.3 Permutasi dengan Pengulangan Sekarang kita akan membahas permutasi di mana sebuah elemen/obyek dapat dipilih lebih dari satu kali. Contoh 12. Ada berapa banyak carakah huruf-huruf dari kata LALALILILU dapat dipermutasi? Jawab: Misalkan kita memberi index pada setiap huruf yang berulang sehingga menjadi L1A1L2A2L3I1L4I2L5U. Maka sekarang terdapat 10 obyek, yang dapat dipermutasi dengan 10! cara yang berbeda. Untuk setiap 10! permutasi ini, huruf A1A2 dapat dipermutasi dengan 2! cara, L1L2L3L4L5 dapat dipermutasi dengan 5! cara, dan huruf I1I2 dapat dipermutasi dengan 2! cara. Jadi sebenarnya 10! terlalu banyak terhitung, dan dapat diperbaiki menjadi ͳͲǨ ʹǨ ͷǨ ʹǨ

Teorema Misalkan terdapat k tipe obyek, dengan: n1 obyek bertipe 1, n2 obyek bertipe 2, dst. Maka banyaknya cara penyusunan dari n1+n2+. . .+nk obyek ini adalah dengan ݊ଵ ൅ ݊ଶ ൅ ‫ ڮ‬൅ ݊௞ ݊ଵ Ǩ ݊ଶ Ǩ ǥ ݊௞ Ǩ

Contoh 13. Ada berapa banyak cara huruf-huruf dari kata SUMSUM bisa disusun sedemikian hingga tidak ada dua huruf yang sama berdekatan? 62

Jawab: Misalkan kita telah memilih SU pada dua posisi pertama, maka susunan yang mungkin bagi huruf M adalah sebagai berikut: S U M M S

U

S

U

M

M M

M

Pada kasus yang pertama, terdapat 2! =2 kemungkinan penyusunan huruf S dan U, pada kasus kedua ada 2!=2 kemungkinan, sedangkan pada kemungkinan ketiga hanya ada 1 kemungkinan. Jadi jika kata yang disusun dimulai dengan huruf S dan U, maka memunculkan 2+2+1=5 kemungkinan penyusunan. Secara umum, kita bisa memilih 3 huruf untuk posisi pertama, dan 2 huruf untuk posisi kedua. Sehingga banyaknya kemungkinan yang dicari adalah 3 ˜ 2 ˜ 5 =30. Latihan 14. Ada berapa banyak bilangan yang bisa dibentuk oleh angka 1, 2, 3, 4, 3, 2, 1 sehingga angka ganjil menempati posisi ganjil? 1.4 Kombinasi tanpa Pengulangan Misalkan diberikan sebuah himpunan dengan n elemen. Pemilihan k anggota himpunan tanpa memperhatikan urutan disebut k-kombinasi, dengan 0  k  n. Jadi k-kombinasi adalah sebuah subhimpunan dengan k anggota (k-subhimpunan). Banyaknya cara untuk memilih r-kombinasi dari sebuah himpunan dengan n anggota ݊ ݊ dinotasikan dengan ቀ ቁ. Dengan demikian ቀ ቁmenyatakan banyaknya k-subhimpunan dari ݇ ݇ ݊ ௡Ǩ himpunan dengan n anggota. Dapat ditunjukkan bahwa ቀ ቁ ൌ ௞Ǩሺ௡ି௞ሻǨ ݇ ݊ ݊ ݊ ݊ Catatan: Perhatikan bahwa ቀ ቁ ൌ ቀ ቁ ൌ ͳ sedangkan ቀ ቁ ൌ ቀ ቁ ൌ ݊. Ͳ ݊ ݊െͳ ͳ Karena n - (n - k) = k , maka untuk untuk bilangan bulat n, k berlaku sifat identitas simetri, yakni ݊Ǩ ݊Ǩ ݊ ݊ ቀ ቁൌ ൌ ൌቀ ቁ ݇ ݊ െ ݇ ݇Ǩ ሺ݊ െ ݇ሻǨ ሺ݊ െ ݇ሻǨ ݇Ǩ

Hal ini bisa diinterpretasikan sebagai berikut: jika ada n pinsil warna yang berbeda, maka banyaknya cara memilih k pinsil sama dengan banyaknya cara memilih n-k pinsil yang tidak diambil. Contoh15.

ͳͳ ͳͳ ቀ ቁ ൌ ቀ ቁ ൌ ͷͷ ͻ ʹ

63

ͳʹ ͳʹ ቁ ൌ ቀ ቁ ൌ ͹ͻʹǤ ͷ ͹



Contoh 16. 2-kombinasi dari himpunan {P, Q, R, S} adalah PQ, PR, PS, QR, QS, RS

Contoh 17. 3-kombinasi dari himpunan {P, Q, R, S} adalah PQR, PQS, PRS, QRS

Contoh 18. Dari 10 buku tulis, kita bisa memilih 4 dengan menggunakan ቀ

ͳͲ ቁ ൌ ʹͳͲ cara. Ͷ

1.5 Kombinasi dengan Pengulangan Teorema (De Moivre) Misalkan n adalah bilangan asli. Banyaknya solusi bilangan asli yang memenuhi x1 + x2 + x · · + xr = n adalah

݊൅‫ݎ‬െͳ ቁ ‫ݎ‬

ቀ Bukti: Tuliskan n sebagai

n =1 + 1 + … + 1,

dengan n angka 1, dan n -1 notasi + . Untuk mendekomposi n menjadi penjumlahan r bilangan kita hanya perlu memilih r – 1 notasi + , yang membuktikan teorema.

Contoh 19. Berapa banyak solusi bilangan bulat non-negatif untuk persamaan x1 + x2 + x3 + x4=11 Jawab: Contoh ini bisa diselesaikan dengan secara langsung menggunakan Teorema De Moivre. Tetapi berikut ini akan diberikan ilustrasi sehingga bisa diperoleh gambaran lebih jelas. Perhatikan barisan dari 14 (11+3) bintang di bawah ini. ************** 64

Pilih tiga dari 14 bintang ini, dan ubah menjadi garis tegak. Dengan demikian, ketiga garis tegak ini membagi 11 bintang menjadi 4 kelompok (sebuah kelompok dimungkinkan untuk tidak memiliki anggota). Setiap pilihan 3 garis tegak berpadanan tepat dengan satu solusi persamaan di atas, dengan memandang banyaknya bintang yang terletak dalam suatu kelompok sebagai nilai dari sebuah variable. Sebagai contoh, susunan di bawah ini berpadanan dengan solusi x1=1, x2=0, x3=5, x4=5. * || *****|*****

Dengan demikian banyaknya solusi dari persamaan di atas adalah ͳͶ ͳͳ ൅ Ͷ െ ͳ ͳͶ ቁ ൌ ቀ ቁ ൌ ቀ ቁ ൌ ͵͸Ͷ ͵ Ͷെͳ ͳͳ



Contoh 20. Tentukan banyaknya lintasan terpendek dari A ke B. B

A

Jawab: Setiap lintasan terpendek dari A ke B harus terdiri dari 9 langkah, di mana 4 di antaranya adalah langkah ke atas dan sisanya langkah ke kanan. Dengan demikian ͻ ͻ banyaknya lintasan terpendek adalah ቀ ቁ ൌ ቀ ቁ ൌ ͳʹ͸Ǥ ͷ Ͷ Contoh 21. Misalkan terdapat sepuluh bola yang identik, dan keranjang yang dinomori 1, 2, …, 8. Tentukan banyaknya cara untuk mendistribusikan bola ke dalam keranjang agar setiap keranjang terisi sedikitnya 1 bola. Jawab: Banyaknya cara adalah

ͳͲ െ ͳ ͻ ቁ ൌ ቀ ቁ ൌ ͵͸. ͺെͳ ͹

ቀ Jelaskan mengapa demikian!

65

2. Prinsip Sangkar Merpati (Pigeon-hole Principle) Prinsip sangkar merpati menyatakan bahwa jika terdapat n barang yang didistribusikan ke dalam n buah kotak, maka sedikitnya satu kotak akan menerima lebih dari satu barang. Untuk menghemat penulisan, pada contoh-contoh berikut prinsip ini ditulis sebagai PHP. Contoh 1. Jika terdapat 13 orang, maka sedikitnya ada dua orang merayakan ulang tahun pada bulan yang sama Contoh 2. kemudian jika rata-rata manusia memiliki dua juta rambut, maka setidaknya ada empat orang di Jakarta memiliki jumlah rambut yang sama di kepalanya. Perhatikan bahwa prinsip sangkar merpati bermanfaat dalam membuktikan masalah eksistensi, yakni kita menunjukkan sesuatu ‘ada’ tanpa perlu diidentifikasi secara kongkret. Contoh 3. (Putnam 1978) Misalkan A adalah himpunan dua puluh bilangan asli yang dipilih dari deret matematika 1, 4, . . . , 100. Buktikan bahwa ada dua bilangan asli berbeda dalam A yang jumlahnya 104. Jawab: Kita partisi ketigapuluh empat anggota dari deret ini menjadi delapan belas himpunan, yakni {1}, {52}, {4, 100}, {7, 97}, {10, 94}, . . . , {49, 55}. Karena kita memilih dua puluh bilangan asli dan kita mempunyai delapan belas himpunan, maka berdasarkan PHP, terdapat dua bilangan yang merupakan anggota dari himpunan yang sama. Perhatikan bahwa jumlah keduanya adalah 104. Contoh 4. Tunjukkan bahwa di antara tujuh bilangan asli positif yang nilainya kurang atau sama dengan 126, bisa kita temukan dua diantaranya, katakan a dan b, yang memenuhi b < a  2b. Jawab: Bagi bilangan-bilangan {1, 2, 3, . . . , 126} menjadi enam himpunan berikut: {1, 2}, {3, 4, 5, 6}, {7, 8, . . . , 13, 14}, {15, 16, . . . , 29, 30}, {31, 32, . . . , 61, 62} dan {63, 64, . . . , 126}. Berdasarkan PHP, dua dari tujuh bilangan pasti terletak dalam salah satu himpunan, dan jelas bahwa kedua bilangan tersebut akan memenuhi ketidaksamaan yang diminta. Contoh 5. Buktikan bahwa bagaimanapun lima puluh lima bilangan dipilih dari {1, 2, . . . , 100}, pasti terdapat dua bilangan yang selisihnya 10. Jawab: Pertama-tama perhatikan bahwa jika kita memilih n + 1 bilangan dari sebarang 2n bilangan asli berurutan, maka terdapat dua bilangan yang selisihnya n. Karena jika kita memasangkan 2n bilangan asli berurutan {a + 1, a + 2, . . . , a + 2n} menjadi n pasang

66

{a + 1, a + n + 1}, {a + 2, a + n + 2}, . . . , {a + n, a+2n}, kemudian kita pilih n + 1 bilangan, maka berdasarkan PHP terdapat dua bilangan berada dalam kelompok yang sama. Sekarang, kelompokkan 100 bilangan menjadi: {1, 2, . . . , 20}, {21, 22, . . . , 40}, {41, 42, . . . , 60}, {61, 62, . . . , 80} dan {81, 82, . . . , 100}. Jika kita memilih limapuluh lima bilangan, maka berdasarkan PHP, pasti terdapat suatu kelompok dimana kita memilih sedikitnya sebelas bilangan. Dengan menerapkan lemma sebelumnya pada kelompok tersebut (yakni untuk n = 10), maka terdapat dua bilangan dengan selisih 10. Contoh 6. (AHSME 1994) Namai sebuah CD dengan label ”1”, dua CD dengan label ”2”, tiga CD dengan label ”3”, . . ., dan lima puluh CD dengan label ”50”. Kemudian letakkan 1 + 2 + 3 + . . . + 50 = 1275 CD berlabel ini ke dalam suatu kotak. Beberapa CD diambil dari kotak secara acak tanpa penggantian. Berapakah jumlah minimum CD yang harus diambil untuk menjamin diperoleh setidaknya sepuluh CD dengan label yang sama? Jawab: Jika kita mengambil semua CD yang berlabel ”1”,. . .,”9” (ada 1+2+. . .+9 = 45 CD) dan sembilan CD dari setiap CD yang berlabel ”10”, . . ., ”50”, maka kita telah mengambil 45+9 ·41 = 414 CD. Maka pengambilan CD ke-415 akan memastikan terdapat setidaknya 10 CD dengan label yang sama. Contoh 7. (IMO 1964) Tujuhbelas orang saling berkorespondensi melalui email. Setiap orang berkorespondensi dengan keenambelas orang lainnya, dan diketahui mereka hanya membahas tiga topik yang berbeda. Jika setiap pasang korespondensi hanya membahas sebuah topik, buktikan bahwa terdapat setidaknya tiga orang saling berkorespondensi dengan topik yang sama. Jawab: Pilih seorang dalam kelompok tersebut, misalkan Candra. Perhatikan bahwa ia berkorespondensi dengan enam belas orang lainnya. Berdasarkan PHP, setidaknya ada enam orang yang berkorespondensi dengan Candra dan membahas topik yang sama, sebut topik tersebut sebagai topik A. Jika di antara enam orang tersebut ada dua orang yang membahas topik A juga, maka masalah selesai. Karena itu berarti Candra dan mereka berdua saling berkorespondensi dengan topic yang sama, yakni topik A. Jika di antara enam orang tersebut tidak ada dua orang yang saling berkorespondensi dengan topik A, maka keenam orang tersebut hanya boleh saling berkorespondensi dengan dua topik lainnya, sebut sebagai topik B dan C. Pilih seorang dari enam orang ini, misalkan Budi. Berdasarkan PHP, di antara lima teman korespondensi Budi ini, pasti terdapat tiga diantaranya yang saling berkorespondensi dengan topic yang sama. Tanpa mengurangi keumuman, misalkan topik B. Jika ada dua dari tiga orang ini yang saling berkorespondensi dengan topik B juga, maka masalah selesai. Tetapi jika tidak, maka setiap dua dari tiga orang ini saling berkorespondensi dengan topik C, dan artinya mereka bertiga saling berkorespondensi dengan topik yang sama, yakni topic C. Terbukti. Contoh 8. Diketahui A suatu himpunan dengan anggota sepuluh bilangan asli yang bernilai di antara 1 dan 99. Buktikan bahwa terdapat dua subhimpunan yang irisannya bukan himpunan kosong dimana jumlah anggota-anggota kedua subhimpunan adalah sama.

67

Jawab: Dari suatu himpunan dengan 10 anggota, kita dapat memperoleh 2101 = 1023 subhimpunan tidak kosong. Untuk setiap subhimpunan ini, kita tentukan jumlah dari anggotaanggotanya. Perhatikan bahwa nilai maksimal yang mungkin kita peroleh dari jumlah anggota suatu subhimpunan adalah 90 + 91 + . . . + 99 = 945 < 1023. Akibatnya, berdasarkan PHP, terdapat setidaknya dua subhimpunan yang memiliki jumlah anggota sama.

Contoh 9. Diberikan 9 bilangan asli sebarang yang faktor primanya anggota dari himpunan {3, 7, 11}. Buktikan bahwa terdapat setidaknya dua bilangan sehingga hasil kalinya merupakan bilangan kuadrat. Jawab: Suatu bilangan asli merupaka,n bilangan kuadrat jika semua pangkat dari faktor primanya merupakan bilangan genap. Perhatikan bahwa semua bilangan asli tersebut dapat dinyatakan sebagai 3a7b11c. Setiap triplet (a, b, c) memiliki salah satu bentuk dari 8 pola paritas berikut: (genap,genap,genap), (genap,genap,ganjil), (genap,ganjil,genap), (genap,ganjil,ganjil), (ganjil,genap,genap), ..., (ganjil, genap, ganjil), (ganjil, ganjil, genap), (ganjil,ganjil,ganjil). Karena terdapat 9 bilangan asli, maka berdasarkan PHP, terdapat dua bilangan yang triplet pangkatnya memiliki paritas yang sama. Pilih dua bilangan ini, maka hasil kalinya merupakan bilangan kuadrat, karena jumlah dari tiap pangkat yang bersesuaian merupakan bilangan genap. LATIHAN Latihan 10. Buktikan bahwa di antara n + 1 bilangan, pasti terdapat setidaknya dua bilangan yang selisihnya habis dibagi n. Latihan 10. Tunjukkan bahwa jika lima titik sebarang semuanya pada atau di dalam suatu persegi dengan panjang sisi 1, maka terdapat sepasang titik yang jaraknya kurang atau sama dengan 2/2. Latihan 12. (Hungarian Math Olympiad, 1947) Buktikan bahwa di antara enam orang dalam suatu ruangan terdapat tiga orang yang saling kenal, atau terdapat setidaknya tiga orang yang tidak saling kenal. Latihan 13. Kita menyebut suatu himpunan ”bebas jumlah” jika tidak ada dua anggota himpunan yang jumlahnya adalah anggota dari himpunan itu juga. Berapa maksimal banyaknya anggota subhimpunan dari himpunan {1, 2, . . . , 2n  1} yang ”bebas jumlah”? Petunjuk: Perhatikan bahwa himpunan {n, n+1,n+2,. . . , 2n-1} dengan n anggota adalah ”bebas jumlah”. Tunjukkan bahwa sebarang subhimpunan dengan n + 1 anggota tidak bebas jumlah. Latihan 14. (MMPC 1992) Misalkan huruf-huruf alfabet dituliskan secara acak. a. Buktikan bahwa pasti terdapat empat huruf konsonan yang berurutan b. Tuliskan suatu daftar huruf alfabet dimana tidak terdapat lima huruf konsonan berurutan. c. Buktikan bahwa jika semua huruf diatur secara melingkar, maka terdapat lima huruf konsonan berurutan. Latihan 15. Misalkan M adalah bilangan asli tujuh belas angka dan misalkan N adalah bilangan yang diperoleh dari M dengan menuliskan angka-angka yang sama tapi dengan 68

urutan terbalik. Buktikan bahwa terdapat setidaknya satu angka dari representasi bilangan M +N yang genap. Latihan 16. Tidak peduli bagaimanapun lima puluh lima bilangan asli dipilih dari {1, 2, . . . , 100}, buktikan bahwa dapat dipilih dua bilangan yang memiliki selisih 9, dua yang memiliki selisih 10, dua yang memiliki selisih 12, dan 2 yang memiliki selisih 13, tetapi tidak harus ada dua yang memiliki selisih 11. Latihan 17. Diketahui mn + 1 bilangan real yang berbeda. Buktikan bahwa terdapat suatu barisan meningkat dengan setidaknya n + 1 anggota, atau barisan menurun dengan setidaknya m + 1 anggota. Latihan 18. Jika titik-titik pada suatu bidang diwarnai dengan tiga warna, tunjukkan bahwa akan selalu terdapat dua titik dengan warna yang sama berjarak satu satuan. Latihan 19. Tunjukkan bahwa jika titik-titik ada suatu bidang diwarnai dengan dua warna, maka akan selalu terdapat sebuah segitiga sama sisi yang titik sudut-titik sudutnya memiliki warna yang sama.

3. Paritas Contoh 1. Misalkan dua persegi di sudut yang berseberangan dari sebuah papan catur dibuang. Tunjukkan bahwa tidak mungkin 62 persegi yang tersisa ditutup oleh 31 domino. Jawab: Setiap domino menutup tepat satu kotak putih dan satu kotak hitam. Tetapi dua kotak persegi yang dibuang berwarna sama, sehingga jelas bahwa tidak mungkin persegi yang tersisa ditutup dengan 31 domino. Contoh 2. Bilangan 1, 2, …, 10 dituliskan dalam suatu baris. Tunjukkan bahwa bagaimanapun tanda penjumlahan atau pengurangan digunakan di depan masing-masing bilangan, maka jumlahnya tidak akan pernah menjadi 0. Jawab: Jumlah dari 1 + 2 + … + 10 = 55, suatu bilangan ganjil. Karena paritas tidak dipengaruhi oleh pemilihan tanda penjumlahan atau pengurangan, maka bagaimanpun tanda di depan masing-masing angka 1, 2, …, 10 dipilih, jumlahnya tidak mungkin menjadi 0. Definisi. Titik latis (m, n) pada suatu bidang adalah titik dengan koordinat bilangan bulat. Definisi. Titik tengah dari suatu ruas garis yang menghubungkan (x,y) ke (x1,y1) adalah titik ‫ ݔ‬൅ ‫ݔ‬ଵ ‫ ݕ‬൅ ‫ݕ‬ଵ ǡ ൰ ൬ ʹ ʹ Contoh3 . Lima titik latis dipilih secara acak. Buktikan bahwa kita selalu bisa memilih dua titik sehingga titik tengah yang menghubungkan keduanya juga merupakan titik latis. Jawab: Perhatikan bahwa terdapat empat pola paritas: (genap, genap), (genap, ganjil), (ganjil, ganjil), (ganjil, genap). Berdasarkan PHP di antara lima titik latis pasti terdapat dua

69

titik yang memiliki paritas yang sama. Pilih dua, jelas bahwa titik tengahnya merupakan bilangan bulat juga. Untuk contoh-contoh berikut, kita perlu mengetahui nama-nama dari tetromino berikut:

Contoh 4. Budi mengambil setiap potong dari masing-masing tetromino pada gambar di atas. Tunjukkan bahwa bagaimanapun penyusunan dilakukan, tidak mungkin dihasilkan sebuah persegi panjang. Jawab: Jika penyusunan sebuah persegi panjang dimungkinkan, maka akan terdiri dari 20 persegi. Warnai persegi-persegi ini sebagaimana pewarnaan pada papan catur. Maka terdapat 10 persegi berwarna hitam dan 10 berwarna putih. Perhatikan bahwa T-tetromino selalu menutupi persegi berwarna hitam dengan jumlah ganjil, sedangkan tetromino lainnya selalu menutupi dengan jumlah genap. Sehingga banyaknya persegi warna hitam yang tertutupi adalah ganjil. Kontradiksi. Contoh 5. Tunjukkan bahwa sebuah papan catur 8×8 tidak bisa ditutupi dengan 15 straighttetromino dan sebuah L-tetromino. Jawab: Warnai baris 1, 3, 5, 7 dengan warna hitam dan baris 2, 4, 6, 8 dengan warna putih. Straight-tetromino akan selalu menutupi persegi putih dengan jumlah genap, sedangkan Ltetromino akan selalu menutupi dengan jumlah ganjil. Jika dimungkinkan ada pengubinan yang diminta, maka kita akan menutupi ubin warna putih dengan jumlah ganjil, kontradiksi. Latihan 6. Dua puluh anak laki-laki dan perempuan duduk di sekeliling meja bundar. Tunjukkan bahwa pasti terdapat seseorang yang diapit oleh dua orang anak perempuan. Latihan 7. Angka 1, 2, …, 2001 dituliskan pada papan. Seseorang menghapuskan sepasang bilangan pada papan kemudian menuliskan selisihnya di papan. Hal tersebut dilakukan berulang kali hingga tinggal tersisa sebuah bilangan. Apakah mungkin bilangan tersebut 0? Latihan 8. Tunjukkan bahwa sebuah papan catur berukuran 10×10 tidak bisa ditutupi dengan 25 straight tetromino. Latihan 9. Tunjukkan bahwa sebuah papan catur berukuran 8×8 tidak bisa ditutupi dengan 15 T-tetromino dan sebuah tetromino. Latihan 10. Sebuah pertemuan dihadiri oleh n peserta. Sejumlah peserta saling berjabat tangan. Tida ada yang berjabat tangan dengan dirinya sendiri dan setiap dua peserta berjabat tangan paling banyak satu kali. Seseorang peserta dikatakan ganjil jika banyaknya jabat tangan yang dilakukannya adalah ganjil. Tunjukkan bahwa banyaknya peserta ganjil adalah genap.

70

Prinsip ini digunakan untuk menentukan kardinalitas dari gabungan himpunan-himpunan yang tidak harus saling lepas. Untuk kasus di mana banyaknya himpunan adalah dua atau tiga, dengan menggunakan diagram Venn kita dapat dengan mudah menunjukkan bahwa |A‫ܤ ׫‬ȁ ൌ ȁ‫ܣ‬ȁ ൅ ȁ‫ܤ‬ȁ െ ȁ‫ܤ ת ܣ‬ȁ,

dan ȁ‫ܥ ׫ ܤ ׫ ܣ‬ȁ ൌ ȁ‫ܣ‬ȁ ൅ ȁ‫ܤ‬ȁ ൅ ȁ‫ܥ‬ȁ െ ȁ‫ܤ ת ܣ‬ȁ െ ȁ‫ܥ ת ܣ‬ȁ െ ȁ‫ܥ ת ܤ‬ȁ ൅ ȁ‫ܥ ׫ ܤ ׫ ܣ‬ȁ. Secara umum, jika diberikan n buah himpunan maka kardinalitas dari gabungan n buah himpunan ini diberikan oleh ௡



ȁ‫ܣ‬ଵ ‫ܣ ׫‬ଶ ‫ ׫‬ǥ ‫ܣ ׫‬௡ ȁ ௡

ൌ ෍ȁ‫ܣ‬௜ ȁ െ ෍ห‫ܣ‬௜ ‫ܣ ת‬௝ ห ൅ ෍ ห‫ܣ‬௜ ‫ܣ ת‬௝ ‫ܣ ת‬௞ ห െ ‫ ڮ‬ሺെͳሻ௡ାଵ ห‫ܣ‬௜ ‫ܣ ת‬௝ ‫ܣ ת ڮ ת‬௡ ห ௜ୀଵ

௜ழ௝

௜ழ௝ழ௞

Contoh 1. Pada sebuah klub olahraga 10 orang menyukai tenis, 15 orang menyukai squash, 12 orang menyukai badminton. Di antara mereka, 5 orang menyukai tenis dan squash, 4 orang menyukai tenis dan badminton, 3 orang menyukai squash dan badminton, dan 2 orang menyukai ketiga olahraga ini. Berapa banyak anggota klub yang menyukai sedikitnya satu dari ketiga cabang olahraga ini? Jawab: Misalkan T, S, B, secara berturut-turut, adalah himpunan anggota klub yang menyukai tenis, squash dan badminton. Maka ȁܶ ‫ܤ ׫ ܵ ׫‬ȁ ൌ ȁܶȁ ൅ ȁܵȁ ൅ ȁ‫ܤ‬ȁ െ ȁܶ ‫ܵ ת‬ȁ െ ȁܶ ‫ܤ ת‬ȁ െ ȁܵ ‫ܤ ת‬ȁ ൅ ȁܶ ‫ܤ ׫ ܵ ׫‬ȁ = 10 + 15 + 12 – 5 – 4 – 3 + 2 = 27 .

71

Related Documents

Diktat Pemograman
June 2020 28
Platos Olimp Beijing
June 2020 2
Pelatihan
July 2020 28
Diktat Urogenitalia
May 2020 29

More Documents from "Syukri La Ranti"